You are on page 1of 69

TABLE OF CONTENTS

Introduction to Evidence...............................................................................................................4
Relevance........................................................................................................................................5
Prejudicial v Probative FRE 403..........................................................................................................5
403 Balancing Test...................................................................................................................5
Myers Four Inference Test........................................................................................................6
Specialized Relevance Rules...................................................................................................................8
Subsequent Remedial Measures.........................................................................................8
Compromise Offers & Payment of Medical Expenses..............................................................9
Liability Insurance..................................................................................................................10
Pleas in Criminal Cases.........................................................................................................11
Character Evidence.....................................................................................................................12
Propensity Rule..................................................................................................................................13
Crimes, Wrongs, or Other Acts.......................................................................................................13
Huddleston Standard..............................................................................................................14
Conditional Relevance..........................................................................................................................15
Mercy rule (proof of defendant and victim’s character)......................................................................16
Habit and Routine................................................................................................................................17
Witnesses......................................................................................................................................18
Competency of Witnesses......................................................................................................................18
Impeachment and Character for Truthfulness....................................................................................18
Impeachment by Opinions, Reputation, and Cross-examination about Past Lies.................18
Impeachment for Past Convictions.........................................................................................20
Impeachment by Prior Inconsistent Statement.......................................................................22
Refreshing Recollection..........................................................................................................22
Hearsay.........................................................................................................................................23
Analyzing a Hearsay Problem................................................................................................24
Hearsay Within Hearsay.........................................................................................................24
Non-Hearsay.........................................................................................................................................24
Opposing Party’s Statements..................................................................................................25
Adoptive Admission................................................................................................................25
Statement by Agents or Someone the Party Authorized..........................................................25
Statements by a Co-Conspirator.............................................................................................25
Declarant-Witness’s Prior Statements...................................................................................26
Hearsay Exceptions- Declarant Unavailable.......................................................................................29
Past Testimony........................................................................................................................29
Statements Against Interest.....................................................................................................30
Dying Declarations.................................................................................................................31
Forfeiture by Wrongdoing......................................................................................................32
Attacking and Supporting the Declarant................................................................................32
Hearsay Exceptions- Regardless of Declarant Availability.................................................................33
Present Sense Impressions, Excited Utterances, and Then Existing Condition (State of Mind)
................................................................................................................................................33
Medical Diagnosis..................................................................................................................34
Recorded Recollection............................................................................................................34
Business Records....................................................................................................................36
Public Records........................................................................................................................37
Hearsay Exception- Residual...............................................................................................................38
Confrontation Clause (6th Amendment)....................................................................................39
Hearsay and The Confrontation Clause..............................................................................................40
Davis/Hammon “Primary Purpose” Test..............................................................................40
Avoiding a Confrontation Clause Problem..........................................................................................41
When Can We Rule Out A Confrontation Clause Issue..........................................................41
Statements That Never Or Usually Do Not Raise A Confrontation Clause Issue..................41
Lay Opinion v Expert Testimony...............................................................................................42
Lay Witness Opinion.............................................................................................................................42
Expert Testimony..................................................................................................................................43
Improper Topics of Expert Testimony....................................................................................44
Basis of Expert Testimony......................................................................................................45
Privileges.......................................................................................................................................47
Steps For Analyzing If There Is A Privilege...........................................................................47
Attorney-Client Privilege......................................................................................................................47
Inadvertent Disclosure Analysis.............................................................................................47
Crime Fraud Exception to Attorney-Client Privilege.............................................................48
Two Marital Privileges.........................................................................................................................48
Spousal Testimonial Privilege................................................................................................48
Marital Confidences Privilege................................................................................................49
Federal rules of evidence.............................................................................................................49
INTRODUCTION TO EVIDENCE

What is the first question to ask on the exam: Does the FRE apply? Check FRE 1101.
Collateral Matters – NO extrinsic evidence
- a collateral matter is relevant to the case solely because it impeaches a witness
Non-collateral Matters – allows extrinsic evidence
- is relevant to the case because it proves a fact in consequence and impeaches a witness. 
● Why do we have evidentiary law?
○ To protect certain social goals/interests, to ensure accurate fact finding, to ensure
trial come to an end, to ensure juries only consider proper evidence (we don’t
trust them fully).
■ Investigating the Mistrust of Juries
● TANNER V. US (1987) [P. 8]
○ Juror came forward saying several other jurors appeared
intoxicated during trial and deliberations. Complaining
juror allowed to testify so conviction can be overturned?
○ We want to ensure that juries deliberate without fear of
oversight; no harassment from lawyers; finality of decision;
preserving public trust of juries. Substance abuse is not an
improper “outside influence” about which juries may
testify under FRE 606(b)
○ Exception: Pena Rodriguez (2017) – exception for
explicitly racist statements
○ A juror may not testify about evidence of jurors’ alcohol
and drug use during a trial.
○ FRE 102 – fairness, efficiency, ascertaining truth, and securing justice
● When do the FREs apply?
○ FRE 101 – proceedings in US courts
■ Go to FRE 1101 for exact courts and exceptions where not applicable
● Types of Evidence
○ Oral Testimony- fact, expert, and character witnesses
○ Real Evidence- physical evidence (i.e murder weapon)You have to lay the
ground work for this. For example authenticate it; this can be done through
distinctive characteristics or through a chain of custody we’re by the
proponent accounts for every individual who handled or possessed it since it
was determined to be relevant to the case
○ Documents
○ Demonstrative Evidence- created by parties to illustrate to jury
○ Stipulations- facts both parties agree are true
○ Judicial Notice- what court notes (i.e court room ID of defendant)
● Direct vs Circumstantial
○ Direct: establishes/disproves a factual issue without requiring trier of fact to make
in inference connecting the evidence with disputed fact
■ Evidence: witness saw defendant rob bank
■ Fact: defendant robbed bank
○ Circumstantial: establishes/disproves a factual issue but requires trier of fact to
make in inference connecting the evidence with disputed fact
■ Evidence: Defendant left town after a robbery
■ Inference: guilty people often flee
■ Fact: Defendant committed robbery
● Evidentiary Related Motions
○ Motion in Limine- motion to admit or suppress a certain piece of evidence prior to
trial
○ Motion to Suppress- defendant alleges that evidence was obtained via
Constitutional violation
● Potential Consequences:
○ Summary judgement
○ Motion under R29 of FRCP
● Objections must be timely and specific.
● When an objection is made, what can a judge do?
○ Sustain, overrule, or provide limiting instructions (FRE 105)
● Two Questions to Ask About Every Piece of Evidence
○ What is the proponent trying to prove with this piece of evidence?
○ Is the offered evidence admissible for the purpose for which the evidence will be
used by the jury?
RELEVANCE
FRE 401- Probativeness + materiality= relevant!
• Ask two questions when determining materiality
o what is the substantive law governing the dispute?
o What is the party’s theory of the case?
FRE 401 * All admissible evidence is relevant, but not all relevant evidence is admissible *
US V. JAMES (1999) [P. 29]

● Where defendant raised a self-defense claim, is victim’s past criminal history relevant?
● It is relevant. Victim’s criminal history is relevant to corroborating the defendant’s testimony
and bolstering her credibility.
● Under the Federal Rules of Evidence, corroborating evidence of a victim’s violent history is
admissible to show the defendant acted in self-defense.
PREJUDICIAL V PROBATIVE FRE 403
FRE 403- always the last step of admissibility analysis (only FRE 609 (a)(2) is exempt from a
403 analysis)

403 BALANCING TEST


• Identify the evidence at issue; is it relevant?
• Determine probative value (the strength of the evidence)
• How strongly does it support that point?
• Examples:
o Confession
o Defendant leaves town
o Fight with victim 10 years earlier
• Identify the danger; is there some other reason it can’t be admitted?
o Hearsay
o Privileged
o Impermissible character evidence
• Balance it out; the judge has discretion; is its probative value outweighed by:
o Unfair prejudice
 the concern is inducing a decision on a basis other than the facts and substantive law governing the dispute
o Confusing the issues/misleading the jury
o Undue delay/waste of time
o Cumulative evidence
• Can you give limiting instructions under FRE 105 rather than exclude it?
• Can you sanitize the evidence?
STATE V. BOCHARSKI (2001) [P. 44]

● Not in dispute that victim died from sustaining multiple stab wounds to the head, dispute
whether defendant was the one who did it. State offered 6 photos depicting the wounds. Were
the photos relevant?

● Facts do not need to be in dispute for the evidence to be relevant, but the purpose for which
the evidence is being offered shall be considered in the analysis. Here, offered to inflame
passions of jury, substantially outweighing any probative value the evidence had.
Inadmissible under RUle 403.
● Evidence which is more prejudicial than probative should be excluded.

US V. JAMES (1999) [P. 54]

● DISSENT: Because the defendant was not aware of the victim’s past charges, it could
not have affected her state of mind. Defense argued (and majority accepted) that the
evidence was for defendant’s credibility, not victim’s character, but the probative value
here was outweighed by its prejudicial effect. The jury’s questions to the judge suggest it
may have been attempting to determine whether Ogden was a “bad man” who ‘deserved
to die,’ which was an impermissible use for the evidence.
US V. MYERS (1977) [P. 58]
- Facts: Where the defendant committed a robbery in Florida on June 13, 1974, and the
defendant ran from an agent (who did not identify himself) at a shopping mall three
weeks later. Then the defendant was in Pennsylvania as of July 29, 1974, and was
suspected of being involved in a robbery there too and was subsequently charged. Then
on August 12, 1974, the defendant “fled” from unidentified officers in California when
he moved several feet away after the officers on the motorcycle intentionally ran into the
car defendant was in on the road. At trial (for the Florida robbery) they used both
instances of flight as evidence of the defendant’s guilt for the Florida robbery.
- Holding: Flight is an admission by conduct. Its probative value, a circumstantial
evidence of guilt, depends upon the degree of confidence with which four inferences can
be drawn: (1) from the defendant’s behavior to flight; (2) from flight to consciousness of
guilt; (3) from consciousness of guilt to consciousness of guilt concerning the crime
charged; and (4) from consciousness of guilt concerning the crime charged to actual guilt
of the crime charged. A jury instruction concerning flight is improper unless the evidence
is sufficient to furnish reasonable support for all four of the necessary inferences.
o The more remote in time the alleged flight is from the commission or accusation
of an offense, the greater the likelihood that it resulted from something other than
feelings of guilt concerning that offense.
- Outcome: Here, not confident in these four inferences. Jury finding they exist required
improper speculation. Evidence of the flight was substantially more prejudicial than
probative because it could not be traced to consciousness of guilt for the Florida robbery.
- Evidence of flight is admissible if the evidence is sufficient to support the inferences that
defendant’s behavior was flight, that he was fleeing because of consciousness of guilt and
thus that he was guilty of the crime charged.
Rule 105
- If the court admits evidence that is admissible against a party or for a purpose — but not
against another party or for another purpose — the court, on timely request, must restrict
the evidence to its proper scope and instruct the jury accordingly.
US V. JACKSON (1975) [P. 80]

● Facts: A bank in New York was robbed on August 23, 1971. Video and eyewitness
accounts were unable to establish the identity of the bank robber. Jackson (defendant)
was arrested in Georgia on November 7, 1971 when a local policeman thought Jackson
was acting suspiciously in an area where an armed robbery had just taken place. Stopped
at a traffic check, Jackson had false identification but no license and was arrested for
driving without a license. Police also found several guns in the car. Jackson later escaped
from a Georgia jail. Jackson was charged with the bank robbery in New York. At his
trial, the prosecution sought to use the evidence that Jackson was in Georgia and used a
fake name as evidence that he was fleeing as result of the robbery. Jackson had also been
indicted for assault in New York in July 1971. Jackson made a pretrial motion to exclude
the evidence that he used a false name when he was arrested in Georgia.

● Issue: Can the court address the problem of highly probative but prejudicial evidence by
having the parties enter into a stipulation concerning the facts that the evidence would
show?

● Holding: Yes. This kind of ruling is justified by Federal Rule of Evidence 102, which
states that the rules of evidence should be administered in a way to ensure a fair
proceeding. Federal Rule of Evidence 403, which provides that relevant evidence should
be excluded if it is more prejudicial than probative, should be read in light of Rule 102
and allows discretionary judgments by the trial court to address highly probative but
prejudicial evidence. Normally, unrelated crimes are not admissible. Because there are
arguments as to the strong probative value and strong prejudicial effect of the fact that
Jackson was in Georgia and using a false name, the evidence regarding the arrest in
Georgia will be excluded if Jackson agrees to stipulate that he was in Georgia shortly
after the robbery and he used a fake name while in Georgia.
● A court may exclude prejudicial evidence that is also of high probative value if the
defendant enters into a stipulation concerning the facts that the evidence would show.
● Stipulation

MYERS FOUR INFERENCE TEST


Evidence of flight is not automatically unfairly prejudicial; it must pass the four-inference test.
1. The defendants conduct was an attempt to flee.
2. A guilty conscience
3. Related to guilt over the charged crime
4. Showed actual guilt for the offense
The stronger the inferences, the stronger the probative value.
o An agreement between parties as to a relevant issue within a legal proceeding such that the
content to which the parties stipulate may serve as the basis for a judicial ruling without the
court needing to weigh evidence relating to such issue.
OLD CHIEF V. US (1997) [P. 82]
 Facts: Old Chief was charged with violation of a federal statute making it illegal for a
convicted felon to carry a firearm. Old Chief had previously been convicted of assault
causing serious bodily injury, a felony. He sought to stipulate to the evidence of his prior
conviction and have the court so instruct the jury. In doing so, he sought to prevent the
prosecution from proffering any other evidence about his prior conviction on account of its
unnecessary prejudicial effect against him in the eyes of the jury. The district court denied
Old Chief’s offer to stipulate and admitted the full record of his prior conviction.
 Issue: Is the full record of a defendant’s prior conviction admissible in a trial for violation of
a statute that requires a felony conviction as one of its elements if the defendant offers to
stipulate to the fact that he has previously been convicted of a felony?
 Holding: No. Evidence is not admissible if its unfair prejudicial effect substantially
outweighs its probative value. Although the prosecution is generally able to prove its case the
way it sees fit, this is not the case where proffered evidence has the tendency to taint the
juror’s minds so that they reach a verdict on an improper basis. In this case, admission of the
full record was error by the trial court because the full record had very limited probative
value, if any, once Old Chief stipulated to the fact that he was a convicted felon. On the other
hand, admission of the full record of an assault conviction could paint Old Chief as a violent
man and a lifelong felon in the eyes of the jury, neither of which would be a proper basis
upon which to render a decision. As a result, the unfair prejudice of the full record of Old
Chief’s prior conviction outweighs its very limited probative value and thus is not
admissible.
 Evidence is not admissible if its unfairly prejudicial effect outweighs its probative value.
SPECIALIZED RELEVANCE RULES
- Each furthers social policies
- Each targets evidence that is unfairly prejudicial
- Most of these rules exclude evidence at issue when offered for some purposes, but not
others

Subsequent Remedial Measures


FRE 407- Subsequent Remedial Measures
• Applies when a plaintiff is injured and there’s a change in policy before trial
• Applies when there’s a change in policy after injury of plaintiff one but would not
apply to plaintiff #2 who is injured after the change in policy but before trial
• Allows evidence for impeachment and if disputed: proving ownership, proving
control, proving feasibility You should prob ping pong these if its debatable
• Applicable to:
• Negligence cases
• Strict liability cases
• Many state evidentiary codes differ and restrict application to fault
based claims
• Contract action cases
TUER V. MCDONALD (1997) [P. 100]
- Facts: Mr. Tuer was to have coronary artery bypass graft surgery. Prior to the surgery,
Mr. Tuer was having chest pains and was prescribed Heparin to help with the pain. As
was protocol at the hospital, he stopped being administered Heparin on the morning of
the surgery because it was not good to have the drug in his blood during surgery. Right
before the surgery, Dr. McDonald (defendant) was called away to an emergency with
another patient. This postponed the surgery. Dr. McDonald considered restarting the
Heparin for Mr. Tuer, but he decided not to because of the danger of having the drug in
the blood during surgery. Approximately four hours later, before the surgery began, Mr.
Tuer went into cardiac arrest. Mr. Tuer died the next day. After Mr. Tuer's death, Dr.
McDonald and the hospital changed the protocol of discontinuing Heparin the morning of
surgeries. Under the new protocol, a patient would receive Heparin right up until being
taken into the operating room. Mrs. Tuer (plaintiff) filed a medical-malpractice suit
against Dr. McDonald. She tried to introduce evidence of the subsequent remedial change
in protocol under either a feasibility or impeachment theory.
- Issue: Does a doctor's professional judgment call to follow an existing medical protocol
render evidence of a subsequent remedial change in the protocol admissible for either
feasibility or impeachment purposes?
- Holding: No. Although proving feasibility or impeachment are exceptions to the general
rule that evidence of subsequent remedial measures is not admissible, a professional
judgment call in not instituting the measure sooner does not trigger those exceptions.
Particularly in the medical context, the concept of feasibility encompasses something
more than just that an alternative is physically possible, because it might be possible to do
nearly anything to a body. A doctor's judgment call should not be read as a broad
pronouncement of the general feasibility of a protocol or course of action; rather, it is a
product of the doctor weighing the risks and benefits of the course of action to the patient
in the doctor's care. Similarly, a doctor's exercise of professional judgment to follow an
existing protocol may not be impeached by evidence of the change in protocol. A
protocol change means simply that risks and benefits of a course of conduct have been
reevaluated; it does not mean that the doctor made an incorrect or dishonest decision to
follow the prior protocol at the time it was still in effect.
- A doctor's professional judgment call to follow an existing medical protocol does not
render evidence of a subsequent remedial change in the protocol admissible for either
feasibility or impeachment purposes. Subsequent remedial measures are inadmissible to
prove negligence, culpability, or a product defect. IT IS ADMISSIBLE TO SHOW
OWNERSHIP AND CONTROL AND FEASIBILITY.

COMPROMISE OFFERS & PAYMENT OF MEDICAL EXPENSES


FRE 408 Compromise offers
To apply:
- Offer or exception
- Conduct or statement during negotiations
- Offered at trial to prove or disprove validity or amount of the claim or to impeach
Triggering evidence: 
- Offers of consideration made to settle a disputed claim or conduct/statements made
during settlement negotiations about the claim*
o Forbidden purpose:  
 To prove or disprove validity or amount of a disputed claim or to impeach
by prior inconsistent statement or contradiction
o Rationales:
 Probative worth may be low; compromise may be motivated by “desire for
peace,” not statement of weakness [Advis. Comm. Notes]
 Social policy of encouraging compromise/settlement of claims
o *except when offered in criminal case and negotiations related to a claim by a
public office exercising its regulatory, investigative, or enforcement authority
BANKCARD AMERICA, INC. V. UNIVERSAL BANCARD SYSTEMS, INC. (2000) [P. 113]
- Facts: Bankcard sued Universal for breach of a credit card servicing contract. Under the
contract, Universal was forbidden to steer merchants it had signed up to competitors of
Bankcard. After Bankcard terminated the contract in March 1993, Universal rolled over
certain accounts to one of Bankcard’s competitors. The reason it gave for doing so was
that it thought it had reached a settlement with Bankcard that allowed such rollovers. The
alleged settlement was never finalized and Bankcard and Universal proceeded to trial. At
the first trial, the judge allowed evidence concerning the settlement talks for the purpose
of explaining why Universal rolled over the accounts. The judge warned Universal not to
go into detail regarding the settlements and not to use the terms settlement or negotiation.
A Universal witness testified that he thought it was permissible to move the accounts
because discussions between the parties’ counsel that led him to believe the accounts
were Universal’s to move. The jury found for Universal at the first trial, but the verdict
was thrown out due to errors, such as the ruling regarding the admission of the settlement
evidence.
- Issue: Is evidence regarding settlement negotiations admissible when it is offered for a
purpose other than establishing or disproving the liability of a party?
- Holding: Yes. FRE 408 prohibits the admission of statements made during settlement
negotiations to prove or disprove liability. The purpose of Rule 408 is to encourage
settlements, and if parties feared that their statements during settlement negotiations
could be used against them, it would discourage settlements. Rule 408 does not however,
prohibit the admission of statements made during settlement negotiations offered for
other purposes. Here, Universal wanted to admit statements made during settlement
negotiations to show why it rolled over the accounts. Although the better course of action
would have been for Universal to wait until a settlement was final before rolling over the
accounts, it would be unfair for the court to exclude the evidence. Excluding the evidence
would not fulfill the purposes of Rule 408 because a settlement is not encouraged if one
party is allowed to seduce the other into violating the contract with promises that its
actions will be allowed as part of the settlement, and then accuse the other party of
violating the contract when the settlement falls through. Universal should be allowed to
bring the settlement for purposes of explaining why it took its actions.
- Evidence regarding settlement discussions, while inadmissible to prove or disprove
liability, is admissible when admitted for other purposes, such as to show why a party
took certain actions.
FRE 409- Payment of Medical Expenses
- Why does FRE exclude a party’s offer to pay medical expenses?
o Excludes only evidence of human impulse such as a driver offering to pay
medical expenses for injured pedestrian
- Triggering Evidence:
o Offers to pay medical, hospital, or similar expenses resulting from injury
- Forbidden Purpose
o To prove liability for the injury
- Rationales
o Probative worth may be low; offer may be motivated by “humane impulses” not
guilt
o Admitting evidence would tend to discourage assistance

LIABILITY INSURANCE
FRE 411- Liability Insurance NOT HEALTH INSURANCE
- Evidence that a person was or was not insured against liability is not admissible to
prove whether the person acted negligently or otherwise wrongfully. But the court
may admit this evidence for another purpose, such as proving a witness’s bias or
prejudice or proving agency, ownership, or control.
WILLIAMS V. MCCOY (2001) [P. 127]
- Facts: Williams was involved in a car accident with McCoy. Williams sued McCoy for
negligence in the accident. Williams went to the emergency room immediately after the
accident and then to a chiropractor four days later. Williams said she did not visit the
chiropractor earlier because he was not available. She also met with the defendant’s
insurance claims adjuster, and after the meeting went badly, hired an attorney. At trial,
defense counsel sought to play up the fact that Williams had hired an attorney before
going to the chiropractor to show that she was a litigious person. The court prevented her
from explaining that the reason she had hired an attorney was because of the negative
experience with the claims adjuster. A jury found McCoy negligent and awarded
Williams $3000. Williams appealed.
- Issue: Is evidence of liability insurance admissible for purposes other than to show
wrongful or negligent conduct?
- Holding: Yes. FRE 411 prohibits the use of evidence concerning insurance used to prove
wrongful or negligent conduct, but such evidence is admissible when used for other
purposes. Williams’ desire to testify concerning the meeting with the claims adjuster did
not go to the question of McCoy’s liability for the accident. It merely explained
Williams’ behavior when defense counsel asked her about hiring an attorney before
visiting the chiropractor. Her answer, without mentioning the claims adjuster in the
timeline, was confusing. Williams should have been allowed to testify concerning her
meeting with the claims adjuster to explain her actions.
- Evidence of liability insurance is not admissible on the issue of wrongful or negligent
conduct but is admissible for other purposes.

PLEAS IN CRIMINAL CASES


FRE 410 pleas, plea discussions, and related statements
● FRE 410 mandatorily applies unless one of the two exceptions is applicable [FRE 410(b)]
● No subjective expectation to negotiate appeal
o Two-part test to determine expectation (US v. Robertson)
▪ Defendant had an actual, subjective expectation of negotiating a plea
▪ The expectation was reasonable given the totality of the objective
circumstances
o Factors Suggesting a Plea Negotiation
▪ Involvement of prosecutor
▪ Involvement of defense counsel
▪ Charges already filed
▪ Specific terms discussed
▪ Government expressed interest in defendant’s offer
● Defendant speaks to their peril to LEOs who merely appear to have authority to negotiate
pleas
● Four triggering pieces of evidence
o Guilty plea later withdrawn
o Nolo contender plea
o Statements in proceedings leading up to withdrawn guilty pleas or nolo contender
plea
o A statement made during plea discussions with prosecuting attorney if discussing
di not result in guilty plea or resulted in a later withdrawn guilty plea
● Forbidden purpose – any use against the defendant in criminal or civil case unless it falls
within an identified exception
o Is another statement made during the same discussion is introduced and “in
fairness the statements ought to be considered together”
o Against the defendant in a criminal proceeding for perjury
● Rationales:
o Probative worth may be low; D may offer to plea eve if not guilty to avoid risk of
trial or a higher penalty
o To encourage plea bargaining
US V. BIAGGI (1991) [P. 138]
● Facts: Six defendants, including Congressman Mario Biaggi and the former CEO of
Wedtech Corporation, John Marriotta were charged with wrongdoing with regard to
Wedteach’s receipt of contracts from the Defense Dept. At his trial, Marriotta was
prevented from introducing evidence that he had declined a deal with the government that
would have given him immunity if he provided information regarding wrongdoing by the
other defendants. Marriotta claimed that such evidence showed consciousness of
innocence because anyone would have taken an offer of immunity if they had any
knowledge of wrongdoing to offer. At trial, the prosecution also presented evidence of
Marriotta’s consciousness of guilt because his wife withdrew a large amount of money
from their joint account and put it in her own name. Marriotta was convicted and
appealed.
● Issue: Is evidence that a defendant rejected an offer of immunity barred by FRE 410?
● Holding: No. FRE 410 excludes evidence of plea negotiations used against defendants,
but it does not apply to evidence of immunity offers that a defendant seeks to use to show
a state of mind consistent with innocence. This sort of evidence is probative in that when
a defendant refuses to testify to others’ wrongdoing when doing so would give him
immunity from prosecution, it is a reasonable inference that he must not have any such
knowledge. A jury could reasonably believe that most people would not reject immunity
if they had knowledge that could grant them immunity and to refuse immunity means that
the defendant has no such knowledge.
● Evidence that a defendant rejected an offer of immunity is not barred by Federal Rule of
Evidence 410.
● US v. Mezzanatto p. 142
Prosecutors may, as a precondition to any plea negotiations, demand that Ds agree that
any statements they make during negotiations may be used to impeach contradictory
testimony they give at trial FRE 410 is waivable

CHARACTER EVIDENCE
Types of character evidence
- Category 1: evidence to prove an individual’s propensity to act in a certain way to prove
conduct (404 a; see Fisher p158 for list of exceptions to the rule against propensity
evidence)
- Category 2: evidence to prove an individual’s character to prove/disprove an element of a
charge, claim, or defense (e.g. defamation)
- Category 3: evidence that reveals an individual’s character but is being used for a
propensity purpose (404b)
- Category 4: evidence to prove a witness has a character for truthfulness or untruthfulness
(608, 609)

Propensity Rule
FRE 404(a) (1) Character Evidence
• (a) Character Evidence
• (1) Prohibited Uses. Evidence of a person’s character or character trait is not
admissible to prove that on a particular occasion the person acted in accordance
with the character or trait.
PEOPLE V. ZACKOWITZ (1930) [P. 147]
- Facts: On November 10, 1929, Mr. Zackowitz shot and killed Frank Coppola on a street
near Zackowitz’s apartment. The issue at trial was not whether Zackowitz was guilty of
murder, but whether the murder was premeditated or an impulsive act. The prosecution
(plaintiff) sought to demonstrate the murder was premeditated, and that Zackowitz was
therefore guilty of first-degree murder. During trial, the prosecution submitted evidence
that Zackowitz owned three pistols and a teargas gun, which were stored in his apartment
at the time of the murder. The trial court ruled that the murder was premeditated and
convicted Zackowitz of first-degree murder
- Issue: Is evidence that serves only to demonstrate a criminal defendant’s propensity
(likelihood) to commit a crime admissible to prove the defendant’s guilt
- Holding: No. Evidence that serves only to demonstrate that a defendant has a propensity
to commit a crime is known as character evidence, and is inadmissible to prove the
defendant’s guilt. This rule of evidence ensures that a jury is not unfairly prejudiced
against a defendant whom they know to have a propensity to commit bad acts. A jury
might otherwise place too much emphasis on the defendant’s prior bad acts rather than on
what the defendant has done in the case at hand, or convict based on the mere fact that
the defendant is a bad person. To avoid such results, courts will generally exclude
character evidence. This is not to say that a defendant’s character has no relevance in a
criminal case. A person with a history of violence may very well be more likely to
commit a violent crime than a peaceful person. However, the high likelihood that such
evidence would unfairly prejudice a jury against a defendant outweighs the minimal
relevance of character evidence.
- Outcome: The trial court permitted the prosecution to admit evidence that Zackowitz
owned several dangerous weapons, none of which were the murder weapon, and all of
which were stored in a box in his apartment at the time of the murder. It is apparent that
the prosecution’s sole aim in admitting this evidence was to demonstrate that Zackowitz,
by virtue of owning such dangerous weapons, is an evil and dangerous man who is
therefore more inclined to have killed the victim with premeditation rather than on
impulse. The evidence has little other relevance to the case. Zackowitz did not use these
weapons to commit the crime, and in fact, did not bring these weapons to the scene of the
crime. To be sure, if Zackowitz had brought these weapons with him, the evidence would
be admissible to show that he prepared for the crime. But this was not the case, and
because the evidence is only relevant to show Zackowitz is a dangerous person with a
propensity to commit the crime with premeditation, the evidence is inadmissible.
Accordingly, the conviction is reversed and a new trial is ordered.
- Evidence that has no relevance other than to demonstrate a criminal defendant’s
propensity to commit a crime is inadmissible to prove the defendant’s guilt.
Crimes, Wrongs, or Other Acts
FRE 404 (b)Crimes, Wrongs, or Other acts
• (1) Prohibited Uses.  Evidence of a crime, wrong, or other act is not admissible to prove a
person’s character in order to show that on a particular occasion the person acted in
accordance with the character.  
• (2) Permitted Uses. Notice in a Criminal Case
• This evidence may be admissible for another purpose, such as proving motive,
opportunity, intent, preparation, plan, knowledge, identity, absence of mistake, or
lack of accident.
• (3) Notice in a Criminal Case

Non-Propensity Uses (Not Exhaustive)


- Motive
- Intent
- Mistake or accident (lack of)
- Identity
- Common plan or scheme
- Knowledge
- Opportunity
US V. TRENKLER (1995) [P. 171]
- Facts: On October 28, 1991, a bomb exploded at the Roslindale, Massachusetts house of
Thomas Shay, Sr., killing one member of the bomb squad and severely injuring another.
Thomas Shay Jr. and Alfred Trenkler were charged with the bombing. Trenkler was tried
separately from Shay Jr. The prosecution contended that Trenkler had built the bomb for
his friend Shay Jr. to use against his father. The prosecution also sought to admit
evidence of a similar bomb built by Trenkler used in a bombing in Quincy,
Massachusetts. The prosecution contended that striking similarities in the two bombs
showed that Trenkler also built the Roslindale bomb. The bombs were similar in
components, design, and construction. The prosecution also brought in testimony that a
computer database of explosives which stored information on bomb characteristics
identified the two bombs as similar. The district court ruled the evidence admissible.
Trenkler was convicted and appealed.
- Issue: Is evidence of other criminal acts admissible to prove that the defendant was the
one to commit the crime?
- Holding: Yes. Under FRE 404(b) evidence of other crimes is admissible for purposes of
proving that the defendant was indeed the person who committed the crime if there is a
high degree of similarity between the crimes. There must be a commonality of
distinguishing features to earmark the crimes as the work of the same person. Here the
bombs were similar in design, characteristics and components. Further, a bombing is in
itself a distinctive method of killing people. The Quincy and Roslindale bombs thus bore
a high degree of similarity. The evidence of the Quincy bomb was admissible to show
that Trenkler also built the Roslindale bomb.
- Evidence of other crimes used to prove identity is admissible if there is a high degree of
similarity between the crimes.

HUDDLESTON STANDARD
• Standard of proof the prosecution must meet in order for an “other act” to be admissible:
A reasonable jury must be able to find by a preponderance of the evidence (more likely
than not) that the “other act” occurred.
HUDDLESTON V. US (1988) [P. 201]
- Facts: Huddleston was charged with possession of stolen property and selling stolen
goods—charges arising out of a shipment of stolen video cassette tapes. The material
issue at trial was whether Huddleston knew the tapes were stolen. The prosecution sought
to introduce evidence of two similar acts under Rule 404(b). The first such evidence was
testimony that Huddleston sold a large quantity of new televisions for $28 each; the other
was testimony that Huddleston was arrested one month after the cassette tape theft for
offering to sell $20,000 of appliances to an undercover FBI agent for $8,000. Huddleston
could not produce a bill of sale for any of the goods and testified that he acquired the
tapes, televisions, and appliances from Leroy Westby. The trial court allowed the
admission of the similar prior acts and Huddleston was convicted of possession of stolen
property.
- Issue: Before evidence of a prior act is submitted to the jury, must a trial court make a
preliminary finding that the prosecution has proved the prior act by a preponderance of
the evidence.
- Holding: No. It is sufficient for evidence of prior acts to be admitted if there is enough
evidence to support a finding by the jury that the defendant committed the prior act.
Requiring the trial court to make a preliminary finding on such a collateral issue
superimposes a level of judicial oversight that is not consistent with Rule 404(b) or its
legislative history. Trial judge shall allow the evidence to go before the jury where a
reasonable jury could reasonably find the conditional fact is true.
- Evidence of a prior act should be admitted under an exception to Rule 404(b) if a jury
could reasonably find that the defendant committed the prior act.
CONDITIONAL RELEVANCE
FRE 104(b) Conditional Relevance
- When the relevance of evidence depends on whether a fact exists, proof must be
introduced sufficient to support a finding that the fact does exist. The court may admit the
proposed evidence on the condition that the proof be introduced later.
o Advisory Committee Notes
o Subdivision (b).
 In some situations, the relevancy of an item of evidence, in the large sense,
depends upon the existence of a particular preliminary fact. […]
Relevance in this sense has been labelled “conditional relevancy.”
Morgan, Basic Problems of Evidence 45–46 (1962).
COX V. STATE (1998) [P. 36]
- Facts: On September 22, 1995, James Leonard was shot and killed while sleeping in his
bed at home. Patrick Cox was accused of the murder. It was alleged that he had killed
Leonard in retaliation because Leonard and his wife had accused Cox’s close friend
Jamie Hammer of molesting their daughter and Hammer was in prison pending the
resolution of those charges. At his trial, the prosecution sought to introduce evidence
concerning what had transpired at Hammer’s bond hearing on the molestation charges,
particularly that Hammer’s bond was not reduced. The prosecution sought to show that
Cox killed Leonard because of the latest developments in Hammer’s case. Cox objected,
arguing that the evidence was inadmissible because it was only relevant if Cox knew
what had happened at the hearing and the prosecution had not shown that. Cox was
convicted of murder.
- Issue: Is evidence whose relevance depends on the fulfillment of a condition of fact
admissible?
- Holding: Yes. FRE 104(b) provides that such evidence is admissible upon, or subject to,
the introduction of evidence sufficient to support to the fulfillment of the condition. The
standard for making such a finding is that the judge “must determine only that a
reasonable jury could make the requisite factual determination based on the evidence
before it.” United States v. Beechum, 582 F.2d 898, 913 (5th Cir. 1978). The court is not
required to weigh the credibility of the evidence or make a finding.
- Evidence whose relevance depends on the fulfillment of a condition of fact is admissible
if the court finds that a reasonable jury could make the requisite factual determination
based on the evidence before it.
FRE 104(a) versus 104(b) in relation to Relevance
¡ FRE 104(a): If there is no factual ¡ FRE 104(b): If there is a factual
dispute conditioning the item’s dispute conditioning the item’s
relevance, a trial judge determines relevance, a trial judge has to
admissibility on a preponderance determine if a reasonable jury
standard. (FRE 401/402 test) could find by a preponderance of
evidence that the conditional fact
exists. 
Factual disputes: FRE 104(a) vs FRE 104(b)
¡ 104(a) disputes: Factual questions ¡ 104(b): Factual question related to
that relate to whether a policy whether a piece of evidence is
reason exists to exclude evidence relevant (i.e., piece of evidence is
relevant if a fact exists but not if a
fact does not exist.)

MERCY RULE (PROOF OF DEFENDANT AND VICTIM’S CHARACTER)


MICHELSON V. US (1948) [P. 234]
- Facts: Michelson was with bribing a federal revenue agent. At trial, Michelson’s counsel
called five witnesses to attempt to prove that Michelson had a good reputation in the
community. On cross examination, the prosecution (plaintiff) asked four of those
witnesses if they ever heard that Michelson was arrested for receiving stolen goods. None
of the witnesses had ever heard this, but the prosecution provided a record to the judge
outside the presence of the jury to show that it had not simply fabricated the arrest to
harm Michelson’s reputation in the eyes of the jury. Michelson was convicted of the
bribery charge.
- Issue: May a prosecutor cross-examine a defendant's character witness about whether he
or she has heard of a defendant's prior arrest or conviction unrelated to the charge for
which the defendant is on trial?
- Holding: Yes. After a defendant has put his character at issue by calling witnesses to
testify to his good reputation, it opens the door to the prosecution cross-examining the
witnesses about that reputation and providing contradictory witnesses. Specifically, the
prosecution may cross-examine a witness about knowledge of the defendant’s prior arrest
after the defendant has put his character at issue, but only for the purposes of refuting
evidence of the defendant’s good character or to test the sufficiency of the witness’s
knowledge about the defendant. Generally, the prosecution may not bring up this type of
character evidence unless it goes to an ultimate issue in the case. However, once the
defendant opens the door by bringing forth positive character evidence, the prosecution
may bring its own evidence as to the reputation of the defendant in the community as
well as for the limited purpose of testing the qualifications of the witness to reliably
testify about the defendant’s reputation. If a witness has not heard at least rumors about
the defendant’s poor character traits, such as an arrest, then it is questionable whether the
witness has sufficient knowledge of his reputation to be reliable.
- A prosecutor may cross-examine a defendant's character witness about whether he or she
has heard of a defendant's prior arrest or conviction unrelated to the charge for which the
defendant is on trial.
FRE 402(a)(2) MERCYRULE
- An exception to the ban on propensity evidence
- Benefits the defendant on the ability to have a defense
- Defendant pays a price – state can attack credibility/character once defendant opens the
door
- Tells us defendant can introduce evidence of defendant’s or victim’s pertinent trait for
propensity purposes
- Requirements
o Criminal case
o Pertinent trait
o Opinion or reputation

FRE 405(a) Methods of Proving Character


- applies to the Mercy Rule / world in which Mercy Rule operates for showing character
- Allows testimony in the form of reputation or opinion on DX; specific instances of
conduct NOT allowed except for on CX
HABIT AND ROUTINE
FRE 406 Habit and Routine Practice
- Difference between habit and propensity evidence comes down to good lawyering
o i.e., character – cleanly, wears gloves, tidy, hygienic // habit – always puts on
gloves when testing
- NOT an exception to the propensity rule because habit evidence concerns that someone
will always respond to a certain stimuli in a specific way
o Different in the (1) specificity of circumstances, (2) regularity of the conduct, and
(3) distinctiveness of the situation producing the conduct.
- Its something below consciousness; its an automated response
HALLORAN V. VIRGINIA CHEMICALS INC. (1977) [P. 253]
- Facts: Halloran was an auto mechanic that used cans of refrigerant to service car air
conditioning units. The refrigerant he used was packaged and sold by Virginia
Chemicals. Halloran was a very experienced mechanic, having serviced hundreds of air
conditioning units using thousands of cans of the refrigerant. When Halloran found it
necessary to accelerate the flow of the refrigerant, which was often the case, he would
heat water in a coffee can and put the can of the refrigerant inside the coffee can to warm
it up. On one occasion, as Halloran warmed up a can of refrigerant, the can exploded,
injuring Halloran. As a result, he brought a products liability action against Virginia
Chemicals. Warnings on the cans of refrigerant cautioned against using an immersion
coil to heat them. Virginia Chemicals sought to introduce evidence that whenever
Halloran warmed up a can of refrigerant, he always used an immersion coil.
- Issue: Is evidence that a plaintiff in a products liability case previously and repeatedly
used a technique cautioned against on the product’s warning label admissible to show
that on the occasion in question he was negligent and ignored the warning label?
- Holding: Yes. Evidence of a person’s habit is admissible to prove that the person acted in
conformity with that habit where the habit involves a deliberate, repetitive pattern of
conduct such that similar future conduct is predictable. The party trying to introduce habit
evidence must provide sufficient evidence to the trial court to determine the behavior is a
habit.
- Evidence of a person’s habitual conduct in certain situations is admissible to prove that
the person acted in conformity with those habits.
WITNESSES
Who can testisfy?
- FRE 601, 602, 605, an 606
How do witnesses testify?
- FRE 603, 604, 611, 613, 614, and 615

How do lawyers impeach witnesses?


- FRE 607, 608, 609, 610, and 613
COMPETENCY OF WITNESSES
Four Competencies to Testify
- Narration (FRE 611)
- Importance of Truth (FRE 603)
- Memory (inferred from FRE 602)
- Perception (inferred from FRE 602)
IMPEACHMENT AND CHARACTER FOR TRUTHFULNESS
Modes of Impeachment
- Noncharacter Impeachment
o Contradiction by Conflicting Evidence
o Contradiction by Past Incosistent Statement
o Evidence of Bias
o The witness does not know the difference between the truth and a lie
- Character Impeachment
o They are by trait a liar
o They have been convicted of a crim (609)
Methods of Impeachment
- Attach narrative capacity
- Attack perception
- Attack memory
- Attack sincerity/integrity
IMPEACHMENT BY OPINIONS, REPUTATION, AND CROSS-EXAMINATION ABOUT
PAST LIES
FRE 404(A)(3) Evidence of a witness’s character may be admitted under Rule 607, 608, and
609
FRE 607 Any party, including the party that called the witness, may attack the witness’s
credibility
FRE 608
- An exception to the bar on propensity evidence
- Only applies to witnesses – defendant could just to testify to avoid these questions
- Allows impeachment by reputation or opinion, or specific evidence (on CX)
- Have to have good faith basis and be probative
- How different than the Mercy Rule [FRE 404(a)(2)]?
o Mercy rule only applies to criminal cases
o Only the defendant opens the door for the mercy rule, either can for 608
o Mercy rule allows propensity evidence on any pertinent trait; 608 only allows
regarding truthfulness/untruthfulness.
Intrinstic vs. Extrinsic Evidence
- Intrinsic: Asking a question of the witness on the stand during cross
- Extrinsic: Bringing in (i) documents,or (ii) testimony by other witnesses to prove the
impeaching point

US V. WHITMORE (2004) [P. 261]


- Facts: Whitmore was charged with one count of unlawful possession of a firearm and
ammunition by a felon and one count of possession of a controlled substance. The
charges were based on a November 1, 2001 police chase. Officer Efrain Soto said that
during the chase, he saw Whitmore throw a gun toward a building prior to being
apprehended. When another officer arrived to assist Soto, the officer found a gun in the
window well of the building toward which Soto said he saw Whitmore throw the gun.
The officers also found a small bag of cocaine base in Whitmore's jacket pocket.
Whitmore's defense at trial was that Soto had fabricated the story about the gun and that
Soto had planted the gun in the window well. In an effort to attack Soto's credibility,
Whitmore attempted to call Jason Cherkis, Bruce Cooper, and Kennith Edmonds as
character witnesses to testify to Soto’s character for untruthfulness. Cherkis was a
journalist who had written a newspaper article about Soto and had interviewed people
who stated that Soto was a liar. The district court excluded Cherkis’s reputation
testimony because Cherkis was not personally acquainted with Soto. Similarly, Cooper
was a criminal-defense attorney who planned to testify that other defense attorneys and
the general court community thought that Soto was a liar. The district court excluded
Cooper’s reputation testimony because Cooper did not know Soto’s reputation within the
entire court community and did not live in Soto’s community. Cooper also planned to
testify to his personal opinion that Soto was a liar, based on an unsupported claim that
Soto had previously lied as a witness against Cooper’s clients. The district court excluded
Cooper’s opinion testimony under Federal Rule of Evidence (FRE) 403, because the
testimony was biased and its probative value was substantially outweighed by its danger
of undue prejudice. Finally, Edmonds lived in a neighborhood that Soto had patrolled as a
police officer until approximately five years earlier. During that time, Edmonds saw Soto
regularly. Edmonds claimed that Soto had wrongfully arrested him approximately six
years prior to Whitmore's trial. The district court excluded Edmonds’s opinion and
reputation testimony based on FRE 403. Whitmore also attempted to impeach Soto's
credibility by cross-examining him about (1) a court's finding that Soto had lied during
his testimony in a previous criminal trial, (2) the suspension of Soto's driver's license, and
(3) Soto's failure to pay child support. The district court did not allow Whitmore to cross-
examine Soto on any of these topics. Specifically, the district court held that the previous
judge's finding that Soto had lied was not probative of Soto's character for truthfulness
because it was only an allegation of misconduct, and the evidence was unduly prejudicial
under FRE 403. The district court prohibited Whitmore from cross-examining Soto on
the other two topics because Whitmore's questioning was based on a state driving-record
document listing the suspension of Soto's driver's license as a result of his failure to pay
child support, and the court found that the document was hearsay. A jury convicted
Whitmore on both counts
- Issue: (1) To offer reputation evidence under Federal Rule of Evidence 608(a), must a
party establish that the character witness is qualified based on acquaintance with the
witness and the witness’s community? (2) Does Federal Rule of Evidence 608(b) allow
an attack on a witness's credibility based on misconduct that tends to show that the
witness is untruthful, even if the misconduct does not result in a criminal conviction?
- Holding: (1) Yes. Under FRE 608, a party may offer evidence of a witness’s character
for truthfulness in the form of an opinion or the witness’s reputation. A witness’s
character for truthfulness is always at issue. To offer reputation evidence, a party must
establish that the character witness is qualified by having an acquaintance with the
witness and the witness’s community. (2) Yes. Under FRE 608(b), a party may attack a
witness's credibility on cross-examination by asking about specific instances of prior
conduct. The prior conduct must be probative of the witness's character for truthfulness,
but the conduct need not have resulted in a criminal conviction. During cross-
examination pursuant to FRE 608(b), the questioner is limited to the witness's answers
regarding the prior conduct and may not prove the prior conduct through extrinsic
evidence. A conviction is not a requirement for a proper subject of cross-examination
under FRE 608(b).
- (1) To offer reputation evidence under Federal Rule of Evidence 608(a), a party must
establish that the character witness is qualified based on acquaintance with the witness
and the witness’s community.
- (2) Federal Rule of Evidence 608(b) allows an attack on a witness's credibility based on
misconduct that tends to show that the witness is untruthful, even if the misconduct does
not result in a criminal conviction.

IMPEACHMENT FOR PAST CONVICTIONS


FRE 609 Impeachment of Witnesses with Prior Convictions
- Very narrow exception to the rule against propensity; just supposed to be used to
ascertain witness’s untruthfulness.
- Allows evidence of past convictions for impeachment: relic from when rules of evidence
barred convicted felons from testifying – believed testimony not credible at all
- Bars the introduction of specific act evidence for purposes other than attacking the
witness’s character for truthfulness
- ONLY applies to convictions – not to charges and not to conduct
- Applies only to witnesses – defendant can avoid by not testifying
- Applies only to convictions offered to attack truthfulness – limiting instructions are
available
- Temporal limits (10 years)
- Juveniles have special protections
o If the witness is not the criminal defendant AND admission is necessary to fairly
determine the defendant’s guilt, can be admitted; otherwise, excluded.
- Rule is more protective of the defendant than 403; probative value must outweigh the
prejudice – not equal. Tilts toward exclusion.
When FRE 609(a) FRE 609(a)(1)(B) FRE 609(a) FRE 609(b)
Prejudicial (1)(A) (felony for (2) [more than 10
Effect . . . (felony for criminal defendant years old]
W other who is W)
than
criminal
defendant)

is less than Admit Admit Admit Admit


probative
value
equals Admit Exclude Admit Exclude
probative
value
somewhat Admit Excludes Admit Exclude
outweighs
probative
value
substantially Exclude Exclude Admit Exclude
outweighs
probative
value
LUCE V. US (1984) [P. 299]
- Facts: Luce was charged with crimes related to his possession of cocaine. He filed a
motion in limine to attempt to exclude evidence of a prior drug possession conviction if
he decided to testify. Luce did not commit to testifying, nor did he proffer what he would
testify about. The United States District Court for the Western District of Tennessee
denied the motion, finding that the evidence of the prior drug possession conviction fell
within the category of admissible impeachment evidence under Rule 609(a)(1). Possibly
because of this ruling, Luce did not testify at trial. The jury found him guilty.
- Issue: Must a defendant testify at trial to preserve his right of review of a trial court’s
denial of his motion to exclude the use of evidence of a prior conviction?
- Holding: To obtain review of a claim of improper impeachment by prior conviction
under Rule 609(a)(1), a defendant must have testified at trial. A reviewing court is not
able to review evidentiary questions like this with no context. Any harm that results from
a district court’s in limine ruling of whether a prior conviction is admissible would be
purely speculative if the defendant never actually testifies because the nature and scope
of the defendant’s testimony would have an effect on the court’s ruling. This is
particularly true where a balancing of the probative value versus the prejudicial effect is
called for, which is the case with Rule 609(a)(1).
- To obtain review of a claim of improper impeachment by prior conviction, the defendant
must have testified at trial.
OHLER V. US (2000) [P. 300]
- Facts: Ohler was charged with importing and possessing marijuana with the intent to
distribute. The prosecution filed a motion in limine, seeking to introduce evidence of
Ohler’s prior conviction for possession of methamphetamine under Federal Rule of
Evidence 609. The district court granted the prosecution’s motion and held that if Ohler
testified at trial, evidence of Ohler's prior conviction would be admissible. Ohler testified
at trial. During direct examination, Ohler preemptively admitted the prior conviction.
Ohler was convicted on both counts.
- Issue: May a defendant who preemptively introduces evidence of a prior conviction on
direct examination claim on appeal that the trial court’s decision to admit the evidence
was error?
- Holding: A defendant who preemptively introduces evidence of a prior conviction on
direct examination may not, on appeal, claim that the trial court’s decision to admit the
evidence was error. After a court rules that the prosecution may introduce evidence of a
prior conviction, the prosecution can still choose whether to actually introduce the
evidence. The prosecution could, for a variety of reasons, decide not to introduce the
evidence. Thus, any potential harm to a defendant by a court’s admission of a prior
conviction is speculative. As a general rule, a defendant who introduces evidence waives
her right to argue on appeal that the evidence should not have been admitted.
- Outcome: Ohler may not argue on appeal that the evidence of her prior conviction was
admitted in error. Ohler chose to preemptively introduce evidence of the conviction on
direct examination, before the prosecution had a chance to admit the evidence on its own.
This choice was within Ohler’s rights, but she may not now claim that the district court’s
ruling on the prosecution’s motion in limine was in error. The district court’s ruling did
not cause the prior conviction to be placed in evidence. Rather, the cause of this evidence
was Ohler’s own testimony.
- A defendant who preemptively introduces evidence of a prior conviction on direct
examination may not claim on appeal that the trial court’s decision to admit the evidence
was error.

IMPEACHMENT BY PRIOR INCONSISTENT STATEMENT


FRE 613
- Collateral – credibility only
o Merely impeach, extrinsic evidence not allowed to prove collateral matters
- Non-collateral – credibility and material fact
o Impeaches and goes to material fact, extrinsic evidence can be introduced to
prove
- 613(a) read in conjunction with 611 sometimes admits prior consistent statements to
rehabilitate witness credibility, but usually used for prior inconsistent statements
US V. INCE (CANVAS)
- Facts: Two performing bands get in a fight at a rap concert, a man fires a fun at trucks
leaving the area. Military police pull over the van carrying Nigel Ince (defendant) Angela
Neumann, and two friends, exiting the area. Two men identified Ince as the gunman.
Neuman gave an unsworn statement to Officer Roger Stevens that Ince admitted to
Neumann that he fired the gun. At trial, prosecution called Neumann to the stand and she
claimed she did not recall Ince’s statement to her and that her unsworn testimony did not
refresh her recollection. Then they called Officer Stevens to testify about what Neumann
told him regarding Ince’s confession.
- Issue: May the prosecution impeach its own witness with otherwise inadmissible hearsay
evidence if that hearsay contains an alleged confession by the defendant?
- Holding: No, FRE 607 provides that the credibility of a witness may be attacked by any
party, even the party calling the witness. One way of impeaching a witness is to show that
she made a prior inconsistent statement. At a criminal trial, however, there are limits to
what the prosecution can do to impeach its own witness. The prosecution may not use
impeachment as an end-run around the rules of hearsay. Because the probative value was
minimal and the prejudicial value high, the trial court should have excluded the evidence.
The prosecution was misuising 613to try and get in their evidence but the court tells us
that 613 cannt be used as a way to get around hearsay.
- When deciding whether to permit the prosecution to impeach its own witness with
otherwise inadmissible hearsay evidence containing an alleged confession by the
defendant, a trial court must determine whether the evidence is more probative than
prejudicial.
US V. WEBSTER (CANVAS)
- Facts: Webster was charged with aiding and abetting a bank robbery. At his trial, the gov
offered testimony of the bank robber (King). Prosecution asked they be allowed to
examine King voir dire because theyd idn’t know what he’d say. This was denied and
king’s testimony exculpated Webster. Prosecturion introduced King’s prior inconsistent
statements to the FBI which INCUPLATED webster as a way of impeaching King.
- Issue: Is evidence of a witness’s prior inconsistent statement admissible for impeachment
purposes in a criminal case under Federal Rule of Evidence 607 if the government offers
the testimony in bad faith as a means to bring in inadmissible evidence?
- Holding: No. The credibility of a witness may be attacked by any party, including the
party who called the witness. However, to impeach its own witness, a party must have
called the witness in good faith and not as a means to introduce otherwise inadmissible
evidence under the guise of impeachment. By seeking to conduct a voir dire of King the
prosecutor showed good faith.
- Evidence of a witness’s prior inconsistent statement is not admissible for impeachment
purposes in a criminal case under Federal Rule of Evidence 607 if the government offers
the testimony in bad faith as a means to bring in inadmissible evidence.

REFRESHING RECOLLECTION
Steps for Refreshing Recollection:
1. Witness says they can’t recall
2. Identify an item and ask witness if that might refresh their memory
3. Show item to opposing counsel
4. Show item to witness
5. Take item away
6. Ask witness: “did that refresh your memory?”
7. Ask witness to testify from “refreshed” memory.
Witnesses may refresh recollection from anything, but only an adverse party can introduce a
writing to refresh recollection.
FRE 612
- Lawyers can help jog a witness’s memory by asking a leading question or refreshing with
statement
- Only allowed to refresh memory, not allowed to be read onto record as hearsay
- Relies on the assumption (fiction) that the statement will make the witness truly
remember and testify from there rather than regurgitating what just read.
LINDSEY V. M&M RESTAURANT SUPPLY (FROM CANVAS)
- Facts: Woman kept notes of workplace discrimination on scraps of paper as they
happened, then later transcribed them into a notebook. That notebook got destroyed but
she had copied the notes from it into a new one before it was destroyed. The Defendant
claimed her reading directly from the notebook is inadmissible hearsay.
- Issue: Can the plainitiff refer to her notebook to refresh her memory? Is it too prejudicial
to the employer?
- Holding: The prejudice to the employer of allowing the employee at trial to refresh her
memory by using the notebook that allegedly contained verbaitim transcrptions of the
original notes as to events of discrimination and her reactions was slight. Thus the
exclusion of notebook, based on employees spoliation of notes she had compiled
contemporaneously was not warranted and the notebook as a later compliation of events
that happended at work and the employees reactions, could not be read directly into
evidence. Court allowed Lindsey to use the notebook to refresh recollection (plus
defendant could have requested limiting instructions).
HEARSAY
FRE 801 Definitions That Apply to This Article; Exclusions from Hearsay
(c) Hearsay. “Hearsay” means a statement that:
(1) the declarant does not make while testifying at the current trial or hearing; and
(2) a party offers in evidence to prove the truth of the matter asserted in the statement.
FRE 802 The Rule Against Hearsay
Hearsay is not admissible unless any of the following provides otherwise:
- A federal statute
- These rules
- Other rules prescribed by SCOTUS

Hearsay Components
- Declarant (not machine or computer, but a person)
- Out of court statement
- Offered for the truth of the matter asserted in the statement

Statements (assertions; nothing is an assertion without intending to be one)


- Oral assertions (note: commands and Qs are generally not considered assertions under
FRE 801)
- Written assertions
- Nonverbal conduct intended as an assertion
o Try to verbalize what the gesture means, if you can’t probably not hearsay
because probably not an assertion.

Preferences for In-Court Statements Exists Because:


- They reduce risks of error
- Credibility can be assessed
- Cross examination can occur
- Declarant takes oath and there is formality

Identifying Statements & Improper Purpose


1. Is the alleged declarant intending to communicatie something?
- No? Then its not hearsay, a statement for hearsay purposes is an intentional assertion. Nothing
is an assertion unless its intended to be one.

ANALYZING A HEARSAY PROBLEM


Is it hearsay? (FRE 801 [a]-[c])
1) Is the witness referring to a statement made by a person?
• The declarant may be the same person as the witness
• A statement must be an intentional assertion
2) Did the declarant make the statement outside of the courtroom (i.e. not on the witness stand)?
3) Is the statement being offered to prove the truth of the matter asserted in the statement, by the declarant?
• Consider:
• What’s the theory of relevancy?  According to the proponent of the evidence, what is this
statement probative of?  
• Is the proponent asking the fact-finder to believe what the declarant intended to assert? 
If 1 – 3 are not satisfied, the evidence is not hearsay and not excluded by FRE 802 (but may be excluded by
other rules). If 1-3 are satisfied, keep going:
1) Does an exclusion (FRE 801[d]) or exception (FRE 803, FRE 804) apply?
- Yes? Then you have a statement for hearsay purposes, so keep going.
2. What are they intending to communicate?
3. Is the thing they are trying to communicate the thing the proponent of the evidence wants the
fact finder to know? (Is the proponent offering the statement “for the truth of the matter
asserted?”

HEARSAY WITHIN HEARSAY


FRE 805
Hearsay within hearsay is not excluded by the rule against hearsay if each part of the combined
statements conforms with an exception to the rule.
NON-HEARSAY
FRE 801 (d) says a statement that meets the following conditions is not hearsay, so statements
that fall within 801(d)(1) or (2) are excluded from the definition of hearsay.

OPPOSING PARTY’S STATEMENTS


FRE 801 (d)(2)(A)
Opposing parties’ statement exclusion is very broad.
- Applies to any statement by a party, whether inculpatory, exculpatory, or neutral
- Statement does not have to be an admission
- Party’s availability is immaterial
Opposing Parties’ Statement Exemption: Two Limits
- Declarant must be a party
- Statement must be offered against the party/declarant

ADOPTIVE ADMISSION
FRE 801(d)(2)(B)
Adoptive Statements
- Party heard and understood the declarant’s statement
- Had the opportunity to respond/was at liberty to respond
- Circumstances naturally called for a response
- Party failed to respond (0r response did not rebut, deny, or demur)

STATEMENT BY AGENTS OR SOMEONE THE PARTY AUTHORIZED


FRE 801(d)(2)(C)-(D)
MAHLANDT V. WILD CANID SURVIVAL & RESEARCH CENTER (1978) [P. 415]
- Facts: Mahlandt brought suit on behalf of his minor son, Daniel, for injuries Daniel
sustained in an alleged attack by a wolf. The wolf was owned by the Wild Canid Survival
& Research Center, Inc. Mahlandt sought to introduce into evidence three statements,
two made by Poos, and one the minutes of a meeting of the Directors of the Center (Poos
did not attend the meeting). The first statement by Poos was a note he left on his boss’s
door that said that the wolf bit a child. The second statement was a verbal statement Poos
made to his boss that the wolf had bitten a child. The minutes of the Center’s meeting
reflect that a significant amount of discussion was held about the legal implications of the
wolf biting a child. The trial court excluded all three statements as hearsay.
- Issue: Are minutes of a meeting of a corporation and statements made by an employee of
the corporation within the scope of his employment admissible against the corporation?
- Holding: Yes. Statements made by a party or its agent within the scope of his agency
about any material fact are admissible against that party if proffered by the opponent.
Similarly, statements made by individuals authorized by a party to make such statements
on the subject in question are admissible against the party. These statements are not
hearsay because they are reliable in that it is highly unlikely that a party, its agent, or an
individual authorized by the party to speak would make a false statement against itself.
- Admissions by an agent of a party within the scope of his agency about any material fact
at issue are admissible against that party.

STATEMENTS BY A CO-CONSPIRATOR
FRE 801(d)(2)(E)
Statements by the co-conspirator, not the defendant
To admit, must show:
- A conspiracy existed at the time the statement was made
- Conspiracy included declarant and the party against whom the statement was offered
- The statement was made during and in furtherance of the conspiracy
BOURJAILY V. US (1987) [P. 421]
- Facts: Greathouse, an FBI informant, arranged to sell cocaine to Lonardo in order for
Lonardo to distribute it to others, including the defendant. Lonardo arranged a meeting in
a parking lot between himself, Greathouse, and the defendant, at which Lonardo would
transfer the cocaine from Greathouse to the defendant. At the meet, Lonardo and the
defendant were arrested and charged with conspiring to distribute cocaine, among other
things. The prosecution sought to introduce into evidence a telephone conversation
between Lonardo and Greathouse recorded prior to the meet, in which Lonardo stated
that he had a “friend” who was interested in buying the cocaine, but wanted more
information. The district court found that a conspiracy existed between Lonardo and the
defendant and also that Lonardo’s statements on the recording were admissible as
statements by a coconspirator. As a result, the court convicted the defendant.
- Issue: May a court use hearsay evidence in making its preliminary determination on
whether a conspiracy exists for purposes of determining the admissibility of the evidence
at trial?
- Holding: Before a coconspirator’s statement is admitted into evidence under the hearsay
exception, a preliminary determination that a conspiracy exists between the declarant and
the defendant must be made by a preponderance of the evidence. In making such a
determination, a court may use the hearsay evidence sought to be admitted under the
coconspirator statement exemption. This is in part because coconspirators’ statements
themselves can be very valuable in determining whether there was in fact a conspiracy.
- In making a preliminary determination of whether the preponderance of the evidence
reveals that there is a conspiracy present for purposes of determining the admissibility of
evidence, a court may use the hearsay statements sought to be admitted.

DECLARANT-WITNESS’S PRIOR STATEMENTS


FRE 801 (D)(1)(A) Inconsistent Statements Offered Substantively
The declarant testifies and is subject to cross-examination about a prior statement, and the
statement:
(A) is inconsistent with the declarant’s testimony and was given under penalty of
perjury at a trial, hearing, or other proceeding or in a deposition;
(B)Past Consistent Statements
(1) A Declarant-Witness’s Prior Statement. The declarant testifies and is subject to
cross-examination about a prior statement, and the statement:
(B) is consistent with the declarant’s testimony and is offered:
(i) to rebut an express or implied charge that the declarant recently
fabricated it or acted from a recent improper influence or motive in so
testifying; or
(ii) to rehabilitate the declarant's credibility as a witness when attacked on
another ground;
(C) Prior Identification
(1) A Declarant-Witness’s Prior Statement. The declarant testifies and is subject to
cross-examination about a prior statement, and the statement:
(C) identifies a person as someone the declarant perceived earlier.

INCONSISTENT STATEMENTS OFFERED SUBSTANTIVELY


FRE 801(D)(1)(A)
(1) A Declarant-Witness’s Prior Statement. The declarant testifies and is subject to
cross-examination about a prior statement, and the statement:
(A) is inconsistent with the declarant’s testimony and was given under penalty of
perjury at a trial, hearing, or other proceeding or in a deposition;

Requirements for Exemption


- Declarant testifies at trial
- Declarant/witness is subject to cross about the prior statement
- Prior statement is inconsistent with courtroom testimony
- Prior statement was given under penalty of perjury
- Prior statement given during proceeding

PAST CONSISTENT STATEMENTS


801 (d)(1)(B)
(1) A Declarant-Witness’s Prior Statement. The declarant testifies and is subject to
cross-examination about a prior statement, and the statement:
(B) is consistent with the declarant’s testimony and is offered:
(i) to rebut an express or implied charge that the declarant recently
fabricated it or acted from a recent improper influence or motive in so
testifying; (STATEMENT MUST BE MADE BEFORE IMPROPER
MOTIVE AROSE) or
(ii) to rehabilitate the declarant's credibility as a witness when attacked on
another ground; (NO TIMING RESTRICTION)
TOME V. US (1995) [P. 454]
- Facts: Tome had primary physical custody of his four-year-old daughter, A.T. A.T.’s
mother had been unsuccessful in her efforts to get full custody. After A.T. spent the
summer with her mother, the mother reported that Tome had sexually abused A.T. Tome
was charged and tried in the United States District Court for the District of New Mexico,
because the events took place on the Navajo Indian Reservation. At trial, Tome defended
himself on the ground that the charges were made up so that A.T. could live with her
mother. A.T. testified, but she was not very forthcoming about what happened. The
prosecution sought to introduce into evidence seven out-of-court statements made by
A.T. to six witnesses describing the alleged sexual abuse. Tome objected, but the
prosecution argued that the evidence was needed to rebut the implicit allegation that A.T.
was lying. The district court admitted the statements under Rule 801(d)(1)(B). The
prosecution placed great weight on the statements during closing arguments but made no
reference to the alleged purpose of rehabilitating A.T.’s testimony. The jury found Tome
guilty.
- Issue: Is a consistent out-of-court statement made by a witness admissible in federal
court to rebut a charge of fabrication or improper motive if the statement is made after the
motive to fabricate arose?
- Holding: Because the statements came after the custody dispute, cannot be introduced
under 801(d)(2)(B) because it was post-motive rather than pre-. If statements are made
after the charge of fabrication or improper motive, their rebuttal is not as strong because
the declarant may have already formed the intent to make false statements. Allowing
these post-charge statements would broaden the rule and shift the focus of trial to out-of-
court statements.
- Under Federal Rule of Evidence 801(d), a consistent, out-of-court statement made by a
witness is admissible to rebut a charge of a fabrication or improper motive, but only if
made before the motive to fabricate arose.

STATEMENTS OF IDENTIFICATION
801(d)(1)(c)
(1) A Declarant-Witness’s Prior Statement. The declarant testifies and is subject to
cross-examination about a prior statement, and the statement:
(C) identifies a person as someone the declarant perceived earlier.
- Requirements: Must be a statement of identification
- Justification: Past statements of ID are considered incredibly reliable because closer to
the actual event
COMMONWEALTH V. WEICHELL (1984) [P. 465]
- Facts: On the night of May 30, 1980, John Foley and several other friends were in Faxon
Park after attending a drive-in movie. They heard four shots and saw a man running out
of a parking lot. One of Foley’s friends screamed and the man turned to look at them, but
continued running. Foley got a full-face look at the man under a streetlight for a full
second. Foley gave a description of the man he saw to the police. Foley also assisted a
police detective in making a composite drawing of the man Foley saw. The detective
assembled a composite, altering features at Foley’s direction until Foley said the
composite looked like the man he saw. The composite was introduced as evidence at
Weichell’s trial. In the courtroom at Weichell’s trial, Foley also identified Weichell as the
man he saw running on May 30. Weichell was convicted of first-degree murder
- Issue: Are composite sketches admissible?
- Holding: A statement of prior identification is not hearsay if it is made by a witness who
testifies at trial and is subject to cross-examination. The statements by Foley that form the
basis for the composite would thus be admissible since Foley was available for cross-
examination. It thus follows that the composite is admissible either because it retains the
character of the statements that led to its creation or because it is not a statement within
the meaning of the hearsay rule. There is no reason why a composite should be
distinguished from statements describing the physical characteristics the witness saw.
- Composite sketches are not hearsay and are admissible.
US V. OWENS (1988) [P. 469]
- Facts: Owens was charged with attempted murder in relation to the assault of a
correctional counselor at a federal prison. The counselor suffered a skull fracture and was
hospitalized for nearly a month. An FBI agent visited the counselor in the hospital a week
after the assault and found that the counselor could not remember the name of his
attacker. After a couple of weeks, the counselor was able to remember the circumstances
of the attack and identified Owens from a photographic array. At trial, the counselor
admitted that he could not remember having seen his attacker. Although the counselor
had been visited by several people while in the hospital, the only visitor he could
remember was the FBI agent. He could not remember whether any visitor might have
suggested that Owens was the attacker. Owens tried to refresh the counselor’s memory
using hospital records indicating that he had identified someone else as the attacker, but
the counselor could not remember having made a different identification. Owens was
convicted.
- Issue: Does the admission of a witness’ testimony about a currently held belief when the
witness cannot remvmer the basis for the belief violate a criminal defendant’s rights
under the confrontation clause? Is testimony concerning a prior, out-of-court
identification of the defendant admissible if the declarant is unable to explain the basis
for that identification due to memory loss?
- Holding: Yes. A prior, out-of-court statement identifying a person after perceiving him is
not hearsay if the declarant testifies at trial and is subject to cross examination concerning
the statement. Thus, if a witness cannot remember a prior identification that he made, the
prior identification is still admissible as long as the opposing party has a chance to cross
examine the witness about the identification.
- (1) The admission of a witness’ testimony about a currently held belief when the witness
cannot remember the basis for the belief does not violate a criminal defendant’s rights
under the Confrontation Clause; and (2) An out-of-court statement identifying a person is
not hearsay if the declarant is subject to cross examination concerning the statement.

HEARSAY EXCEPTIONS- DECLARANT UNAVAILABLE


FRE 804(a) gives criteria for being Unavailable
Determining whether a witness is unavailable under 804 is a 104(a) question and the burden of
proving unavailability lies with the party trying to enter the statement.
Privilege Requires that an actual claim be made and that the court sustains
804(a)(1) the claim – e.g., spousal, 5th amend.
Refusal to Testify Requires declarant to take the stand and refuse to testify despite
804(a)(2) the threat of contempt or other judicial pressure – e.g., invokes
privilege, but denied
Lack of Memory Requires declarant to take the stand, and requires proponent to
804(a)(3) make good-faith effort to get witness to testify (including
refreshing memory under Rule 612)
Death/infirmity Need not be a permanent infirmity; has been invoked for
804(a)(4) infirmities as short as one or two weeks in duration
Unable to Procure Must take reasonable steps to procure declarant’s attendance
804(a)(5) (and testimony), including a request to appear voluntarily
FRE 804 (B) provides exceptions to the hearsay rule when the declarant is unavailable and it
involves:
- Former testimony
- Statements against interest
- Statements made under the belief of imminent death
- Statements of personal or family history
- Statements offered against a party that wrongfully caused the devlarant’s unavailability

PAST TESTIMONY
804(b)(1)
Rationale
- Because the testimony was given under oath, we assume it is reliable
- Former testimony necessarily allowed for cross examination
Requirements
- Declarant is unavailable
- Prior statement was given at a trial, hearing, or deposition
- Opponent [or predecessor in interest] had opportunity to develop (challenge) testimony
- Opponent had similar motive to develop (challenge) testimony
US V. DUENAS (2012) [P. 475]
● Facts: On April 19, 2007, Guam police officers executed a search warrant at the home of
Ray Duenas and Lou Duenas and seized narcotics, guns, and drug ledgers. The
defendants were arrested and taken to the precinct. Officer Frankie Smith obtained a
waiver of Ray’s Miranda rights and obtained a statement from Ray admitting that he sold
methamphetamine in exchange for stolen items. Ray was charged with conspiracy to
distribute over 50 grams of methamphetamine. Ray later moved to suppress his statement
on grounds that his statement was involuntary and obtained in violation of Miranda. At
the suppression hearing, Officer Smith testified about Ray’s statement. The district court
denied Ray’s motion. Officer Smith was subsequently killed in an accident. The district
court deemed him unavailable at trial and admitted his suppression hearing testimony.
The jury convicted Ray and sentenced him to 25 years in prison.
● Issue: Is former testimony at a hearing by an unavailable witness admissible at trial if it is
offered against a party who had an opportunity and similar motive to develop the
testimony through direct, cross-, or redirect examination?
● Holding: Yes. Former testimony at a hearing by an unavailable witness is not hearsay if
it is offered against a party who had an opportunity and similar motive to develop the
testimony through direct, cross-, or redirect examination as he would have at trial.
● Outcome: Here, Ray’s motive to cross-examine Smith at the suppression hearing was to
show that his statement to Smith was involuntary and in violation of Miranda. Thus,
Ray’s counsel at the hearing only questioned Smith about the circumstances surrounding
the arrest and statement. At trial, Ray’s motive for cross-examining Smith would have
been to challenge the substance of Smith’s statements. The voluntariness of his statement
or his Miranda rights would not even be at issue at trial. Therefore, Ray did not have a
similar motive to develop Smith’s testimony, and his conviction must be reversed.
● Former testimony at a hearing by an unavailable witness is admissible at trial if it is
offered against a party who had an opportunity and similar motive to develop the
testimony through direct, cross-, or redirect examination.

STATEMENTS AGAINST INTEREST


FRE 804(b)(3)
Requirements
- Declarant unavailable
- Statement against declarant’s interest in one of three ways
o Contrary to proprietary or pecuniary interest
o Renders invalid a claim she had against another person
o Exposes the declarant to civil or criminal liability
- Against declarant’s interest at the time the statement was made
- Corroboration for statements against penal interest when offered in a criminal case

WILLIAMSON V. US (1994) [P. 489]


● Facts: After being pulled over by law enforcement, Harris was arrested when cocaine
was found in his rental car. While in custody, Harris admitted to transporting the cocaine
for Williamson. Specifically, Harris told a DEA agent that he was taking the cocaine to
Atlanta for Williamson, who was traveling in front of Harris in another rental car at the
time Harris was stopped. Harris indicated that he could not make a controlled delivery of
the drugs to Williamson because Williamson had seen the police stop Harris and search
his vehicle. Williamson was subsequently charged with cocaine-related offenses. At trial,
Harris refused to testify against Williamson, but the judge allowed the DEA agent to
testify about Harris's confession under the hearsay exception for statements against
interest, because Harris had implicated himself in his statements and was unavailable to
testify. Williamson was convicted on the cocaine charges
● Issue: Does the declarations-against-interest exception to the hearsay rule permit the
introduction of non-self-inculpatory statements made as a part of broader statement that is
generally against the declarant’s interest?
● Holding: No. Non-self-inculpatory statements are inadmissible under the declarations-
against-interest exception to the hearsay rule. This is true even if a non-self-inculpatory
statement is made as a part of broader statement that is generally against the declarant’s
interest. The non-self-inculpatory statements are not trustworthy enough to be admitted
under a hearsay exception, because declarants often have a motive to implicate the
defendant in order to exculpate themselves.
● Outcome: Harris's confession to the DEA agent implicated Harris in the possession of
the cocaine. However, this does not mean that the entirety of Harris's confession was
properly admitted. Harris’s self-inculpatory statements are clearly against his own
interest and admissible, but his statements that tended to implicate Williamson are not as
clearly against his interest. Harris may have had incentive to implicate Williamson to
decrease his own exposure to criminal liability.
● The declarations-against-interest exception to the hearsay rule does not permit the
introduction of non-self-inculpatory statements even when made as a part of broader
statement that is generally against the declarant’s interest.

DYING DECLARATIONS
FRE 804(b)(2)
(b) The Exceptions. The following are not excluded by the rule against hearsay if the declarant is
unavailable as a witness:
***
(2) Statement Under the Belief of Imminent Death. In a prosecution for homicide or in a
civil case, a statement that the declarant, while believing the declarant’s death to be
imminent, made about its cause or circumstances.

Requirements
- Declarant is unavailable
- Only available in homicide prosecutions and civil cases
- Declarant believes death is certain and imminent
o Subjective standard. Factors to consider:
 Declarant’s conduct or statements (e.g., “Get a priest!”)
 Obviousness of declarant’s condition
 Statements to declarant by others (e.g., doctors)
- Statement must relate to the cause or circumstances of death
- Personal knowledge of the “acts that are delclared”

SHEPHARD V. US (1933) [P. 499]


- Facts: Shepard was charged with murdering his wife. In proving the charge, the
prosecution sought to introduce into evidence a statement by Shepard’s wife to her nurse
stating that her husband had poisoned her. Based on this evidence, Shepard was
convicted of murder.
- Issue: Is an out-of-court statement made by a declarant who is not in imminent
expectation of death admissible under the dying-declaration exception to the hearsay
rule?
- Holding: For a dying declaration to be admissible as an exception to the hearsay rule, the
person making it must expect to die soon and have no hope of recovery. In addition, the
declaration must be made with knowledge of facts, not mere suspicion or conjecture.
- Outcome: Here, Shepard’s wife did not appear to believe that her death was imminent
when she told the nurse that Shepard had poisoned her. In fact, she appeared to be
recovering and expressed to the doctor her hope that she would get better.
- An out-of-court statement made by a declarant who is not in imminent expectation of
death is not admissible under the dying-declaration exception to the hearsay rule.
**Remember FRE 602 Need for Personal Knowledge
A witness may testify to a matter only if evidence is introduced sufficient to support a finding
that the witness has personal knowledge of the matter.  Evidence to prove personal knowledge
may consist of the witness’s own testimony.  This rule does not apply to a witness’s expert
testimony under Rule 703.

ACN to FRE 803: “In a hearsay situation, the declarant is, of course, a witness, and neither this
rule nor Rule 804 disposes with the requirement of firsthand knowledge.” 

FORFEITURE BY WRONGDOING
804(b)(6)
Rationale- Justified by necessity, reliability, and fairness (don’t want defendant to benefit from
wrongdoing)
Requirements
- Opposing party must have “wrongfully” cause or acquiesced in the witness’s
unavailability
- Opposing party must have intended to make the declarant unavailable as a witness
- The wrongdoing must have caused the declarant to become unavailable
US V. GRAY (2005) [P. 505]
- Facts: Gray was accused of killing her first husband, her second husband, and her former
boyfriend. Gray was indicted on charges of mail and wire fraud relating to her receipt of
insurance proceeds following the deaths of Robert Gray and Goode. At trial, the district
court admitted several of Robert Gray's out-of-court statements concerning threats that
Gray had made against him and previous assaults on him by Gray and Goode. The
evidence included Robert Gray’s criminal complaint against Gray for assaulting him. The
district court admitted Robert Gray's statements pursuant to Federal Rule of Evidence
804(b)(6), which provides an exception to the hearsay rule when a defendant’s own
misconduct renders a declarant unavailable as a witness for trial. Gray argued that FRE
804(b)(6) shouldn’t apply because she did not kill the declarant to keep him from
testifying at this trial.
- Issue: Does a defendant who wrongfully and intentionally renders a declarant unavailable
as a witness in any proceeding forfeit the right to exclude, on hearsay grounds, the
declarant's statements at that proceeding and any subsequent proceeding?
- Holding: Yes. A defendant who wrongfully and intentionally renders a declarant
unavailable as a witness in any proceeding forfeits the right to exclude, on hearsay
grounds, the declarant's statements at that proceeding and any subsequent proceeding.
FRE 804(b)(6) is a protective rule, and ruling in defendant’s favor here would undermine
the rule’s intent. Rule applies whenever the defendant cause the declarant to be
unavailable where part of the motive was to keep them from testifying in a proceeding
(not limited to this proceeding).
- A defendant who wrongfully and intentionally renders a declarant unavailable as a
witness in any proceeding forfeits the right to exclude, on hearsay grounds, the
declarant's statements at that proceeding and any subsequent proceeding.

ATTACKING AND SUPPORTING THE DECLARANT


When a hearsay statement — or a statement described in Rule 801(d)(2)(C), (D), or (E) — has
been admitted in evidence, the declarant’s credibility may be attacked, and then supported, by
any evidence that would be admissible for those purposes if the declarant had testified as a
witness. The court may admit evidence of the declarant’s inconsistent statement or conduct,
regardless of when it occurred or whether the declarant had an opportunity to explain or deny it.
If the party against whom the statement was admitted calls the declarant as a witness, the party
may examine the declarant on the statement as if on cross-examination.
HEARSAY EXCEPTIONS- REGARDLESS OF DECLARANT AVAILABILITY

PRESENT SENSE IMPRESSIONS, EXCITED UTTERANCES, AND THEN EXISTING


CONDITION (STATE OF MIND)
FRE 803(1), (2), and (3)
Present Sense Impression Excited Utterance
As impression occurs or While excited by event
immediately thereafter
Description, no analysis Relates to event; may include
analysis
Rationale for Then Existing Condition (State of Mind)
- reliability; statement speaking to declarant’s own then existing state of mind
MUTUAL LIFE INSURANCE CO. V. HILLMON (1892) [P. 515]
- Facts: Hillmon brought suit against Mutual Life to recover on her husband’s life
insurance policy. There was a question of whether a body found in Crooked Creek was
actually her husband’s or that of another man, named Walters. In attempting to prove that
Walters accompanied Sallie’s husband on his trip to Crooked Creek and so could have
been the body recovered, Mutual Life sought to introduce into evidence two letters from
Walters to his sister and fiancé sent before Sallie’s husband’s trip to Crooked Creek. The
letters expressed Walters’s intent to accompany Sallie’s husband on the trip. The trial
court excluded the letters as hearsay.
- Issue: Are out-of-court statements indicating a declarant’s intent to do something
admissible?
- Holding: Yes. Out-of-court statements that show the declarant’s intentions are admissible
to prove such intentions if the act intended is material to an issue of the case. Such a
statement of a then-existing intention is just as trustworthy, if not more so, as the
declarant’s own testimony regarding his prior intentions would be.
- Outcome: in this case, the letters are admissible to prove Walters’s then-existing intent to
join Sallie’s husband on his trip to Crooked Creek. The letters do not prove that Walters
actually went to Crooked Creek, merely that he had the intention to do so. This is relevant
because it makes it more probable that he did actually go to Crooked Creek and thus
more probable that the body found was his.
- Out-of-court statements are admissible to prove the declarant’s intent to perform a
particular act when that act is at issue in the case.

SHEPARD V. US (1933) [P. 520]


- Facts: Shepard was charged with murdering his wife. In proving the charge, the
prosecution sought to introduce into evidence a statement by Shepard’s wife to her nurse
stating that her husband had poisoned her. Based on this evidence, Shepard was
convicted of murder.
- Holding: State of mind must go to the declarant’s state of mind, not that of another.
- Outcome: The Court of Appeals admitted the statement under the state-of-mind
exception, as it was offered to rebut the defense’s theory that the wife had committed
suicide. Aside from the fact that it was improper to admit evidence offered for one
purpose at trial for another purpose on appeal, the prosecution did not use the declaration
as proof of the wife’s thoughts or feelings but of someone else’s actions. This was an
improper use of hearsay evidence.

MEDICAL DIAGNOSIS
FRE 803(4)
Rationale
- We think it is more reliable because statements made for want of proper medical
treatments so less likely to lie to one’s doctor.
Requirements
- Statement made for a medical diagnosis or treatment (objective)
- Statement must be reasonably pertinent to diagnosis or treatment (objective)
- Statements must fit within one of the three following categories
o Accounts of medical history
o Descriptions of past or present symptoms or sensations
o Reports about the “inception” of the condition or its “general cause”
- Allows for statements made to physicians, medical assistances, or family members where
seeking medical treatment
- Ordinarily, statements indicating fault do not come in
o Sometimes important for treatment and allowed in (child abuse, domestic
violence, etc.) where the perp ID is needed for the “treatment plan” – separating
victim from perp
US V. IRON SHELL (1980) [P. 532]
- Facts: Iron Shell was accused of assault with intent to rape a nine-year-old girl, Lucy.
Lucy was examined by Dr. Mark Hopkins the night of her assault. During the course of
his examination of Lucy, Hopkins asked her what happened, and Lucy replied that a man
had dragged her into the bushes. Hopkins then asked her if the man had taken her clothes
off. Lucy replied that he did. At Iron Shell’s trial, the prosecution called Hopkins and
Hopkins testified as to what Lucy had said during the examination over Iron Shell’s
objections (said not pertinent to treatment). Hopkins explained that he used Lucy’s
answers to determine where on Lucy’s body to examine more closely. The prosecution
claimed that the statements fell under the hearsay exception in Rule 803(4), that they
were statements made for the purposes of medical diagnosis or treatment or the cause of
medical symptoms reasonably pertinent to diagnosis or treatment. The court permitted the
testimony. Iron Shell was convicted.
- Issue: Are out-of-court statements describing the cause of medical symptoms and which
are reasonably pertinent to diagnosis or treatment admissible?
- Holding: Yes, the “reasonably pertinent” requirement is very broad. Where the doctor
solicits the information from the patient, we assume it is pertinent.
- Statements made out-of-court are admissible if they are made for purposes of medical
diagnosis or treatment, describe medical history or past or present symptoms, or the cause
of symptoms insofar as they are reasonably pertinent to diagnosis or treatment.
RECORDED RECOLLECTION
FRE 803(5)
Rationale
- Deem recorded recollections more accurate because closer in time to the perceived event
Requirements for Admissibility
- Witness must have firsthand knowledge of the event
- The out of court statement must appear in a record
o FRE 101(b)(4) – records include a memo, report, or data compilation
- Testifying witness must no longer recall the information contained in the record “well
enough to testify fully and accurately”
- Witness must vouch for the accuracy of the prior statement/record
*the record itself does not come into evidence (unless an adverse party requests). Witness reads
the document into the record.*

JOHNSON V. STATE (1998) [P. 543]


- Facts: Johnson was accused of murdering Frank Johnson, Jr. At trial, the prosecution
presented a statement made by an eyewitness to the murder, Reginald Taylor. The
statement was made around the time of the murder and was signed by Taylor. At trial,
Taylor did not cooperate with the prosecution when he appeared as a witness. Taylor
claimed he did not remember giving the statement or what he said to the police or what
happened the day of the murder. Taylor admitted his recollection of the murder was
fresher when he gave the statement and admitted that it was his signature on the
statement. The trial court permitted the statement to be read in its entirety to the jury.
Johnson was convicted, sentenced to death and appealed.
- Issue: Is a past recorded recollection of a witness admissible under FRE 803(5) if the
witness does not vouch for the accuracy of the written statement?
- Holding: No, one must vouch for accuracy of the written memo. An assertion in the
statement that it is accurate or a previous acknowledgement under oath is not sufficient as
these statements are not self-verifying. To get a recorded recollection in, must vouch for
the accuracy of the written memo by:
o Witness testifying that she presently remembers recording the fact correctly or
remembers recognizing the writing as accurate when she read it at an earlier time
o Witness testifying that she knows the memo is correct because of a habit or
practice to record matters accurately or check them for accuracy
o Witness testifying to recognizing her signature on the statement and believes it to
be accurate because she wouldn’t have signed it if she had not believed it to be
true at the time.
- A past recorded recollection of a witness is admissible under Rule of Evidence 803(5) if
the witness had firsthand knowledge of the event, the written statement was made at or
near the time of the event, the witness lacks a present recollection of the event, and the
witness vouches for the accuracy of the written statement

REFRESHING MEMORY COMPARED TO RECORDED RECOLLECTION


BUSINESS RECORDS
FRE 803(6) and (7)
Four Easy Steps
- A “business” and some records
- A qualified witness or certification to lay foundation, including:
o Record was made by a person with knowledge or transmitted to record keeper by
one with personal knowledge
 THIS DOES NOT INCLUDE Information transmitted from a customer
or other business outsider to the business if offered for its truth, even if the
business collects that information as part of its regularly conducted
business activity.
o Near the time
o In the course of a regularly conducted business activity
o It was the business’s regular practice to make the record
o Alternatively, the proponent of the evidence can offer a certified record by the
custodian of the record or otherwise qualified person [FRE 902(11) and (12)]
- Regularity requirement
- Rebut any showing that circumstances indicate a lack of trustworthiness
PALMER V. HOFFMAN (1943) [P. 553]
- Facts: Plaintiff brought a negligence suit against a railroad company for the death of his
wife and his own personal injuries resulting from an accident. The railroad company
sought to introduce into evidence a statement made by the engineer of the train (who had
since died) as part of a post-accident interview conducted at the railroad company’s
office. The railroad company regularly conducted interviews of its employees after they
got into accidents.
- Issue: Is a statement made in an employer’s post-accident interview by an employee
involved in the accident admissible as a statement made in the regular course of business
for a railroad company?
- Holding: No. A business record is admissible if made in the regular course of business
and it is the regular course of business to make such a record. However, the fact that a
company regularly makes a record in certain situations does not necessarily mean that
that record is the regular course of business as contemplated by the hearsay exception.
The record must actually be made for the “conduct of the business as a business.” Thus,
although the railroad company always interviews those employees that get into accidents,
this conduct is still not within the regular course of business for a railroad company.
Unlike recording transactions, payrolls, trip routes, etc., interviewing employees after an
accident is not a systematic routine of the railroad business. The interview was conducted
under unusual circumstances, in preparation for litigation, and thus tainted with
untruthfulness, not meeting FRE 803(6)(E).
- A business record is admissible if made in the regular course of business and it is the
regular course of business to make such a record.
US V. VIGNEAU (2000) [P. 556]
- Facts: Vigneau was charged with money laundering. At trial, the prosecution sought to
introduce into evidence several Western Union “To Send Money” forms in their entirety.
The forms contained Vigneau’s name as the sender. At the time, Western Union did not
require proof of the sender’s identity. The district court admitted the forms in their
entirety, including Vigneau’s personal information, and convicted Vigneau.
- Issue: Is a sender’s personal information on a bank form admissible under the business
records exception to the hearsay rule when that information was not independently
verified?
- Holding: No, information provided by outsiders cannot come in under FRE 803(6). The
source of the information contained in the business record must be trustworthy. Under
Johnson v. Lutz, the business records exception does not apply to statements within a
business record that are made by an individual who is not a part of the business. In this
case, whoever wrote Vigneau’s name on the Western Union form (even if it was in fact
Vigneau) was not a part of Western Union’s business. And because the sender’s identity
was not verified, the information is not trustworthy because it theoretically could have
been anyone who wrote Vigneau’s name on the form.
- Outcome: The Western Union form itself is admissible as a business record, but not in its
entirety. The parts of the form with Vigneau’s unverified personal information should
have been redacted.
- To be admissible under the business records exception to the hearsay rule, the source of
the information in the record must be trustworthy.

PUBLIC RECORDS
FRE 803(8)
Admissible as a public record?
- Make sure there is a public record
o FRE 101(b)(3) defines public office
o FRE 101(b)(4) defines record
- If not public, must satisfy 803(6) to get in
- Records of office’s activities are generally admissible [FRE 803(8)(A)(i)]
- Observations pursuant to legal duty to report are generally admissible [FRE 803(8)(A)
(ii)]
o Except matters observed by law enforcement personnel; law enforcement
observations are not admissible against criminal defendants because of concern
that law enforcement observations might not be truthful
 Some courts have construed FRE 803(8)(A)(ii) as not barring a defendant
from introducing law enforecement personnel observations, just the
prosecution
- Factual findings from a legally authorized investigation of a public office are admissible
in a civil case and against the government, but NOT ADMISSIBLE against a criminal
defendant [FRE 803(8)(A)(iii)]

What is the rationale for public records as admissible hearsay?


- Assumption that public officials are honest and diligent and therefore their records
reliable.
- Necessity – because of so many records, hard to remember details independently of the
record.

How different than business records exception [FRE 803(6)]?


- 803(8) allows for one off investigation by public officials/agencies Doesn’t have to be
routinely generated like business records.

Trustworthiness Requirement
- To determine trustworthiness, consider:
o The timeliness of the investigation
o The investigators skill or experience
o Whether a hearing was held
o Possible bias when reports are prepared

BEECH AIRCRAFT CORP. V. RAINEY (1988) [P. 562]


- Facts: Two pilots died in the crash of a Navy training aircraft. The cause of the accident
was unknown. The pilots’ spouses brought a products liability suit against Beech Aircraft
Corp., the manufacturer of the plane that crashed, on a theory that the crash was caused
by the plane’s engine failure. Beech defended itself on the theory of pilot error. Beech
sought to introduce into evidence an investigative report prepared by Lieutenant
Commander William Morgan pursuant to an order from the training squadron’s
commanding officer. The report was broken down into sections labeled “finding of fact,”
“opinions,” and “recommendations.” Specifically Beech sought to introduce a statement
in the “opinions” section which stated that “[t]he most probable cause of the accident was
the pilots failure to maintain proper interval.”
- Issue: Are conclusions and opinions made pursuant to a public agency’s investigation
admissible?
- Holding: Yes. Conclusions and opinions made pursuant to a public agency’s
investigation it was required to make are admissible as long as the conclusion or opinion
is based on the factual findings from investigation. The conclusion or opinion must also
satisfy a trustworthiness requirement built into Rule 803(8). This is because “factual
findings” as written in Rule 803(8)(C) should not be read to literally mean “facts.” A
finding of fact may also be a conclusion based on facts. Furthermore, Rule 803(8)(C)
does not actually state that factual findings are admissible, but rather that “reports . . .
setting forth . . . factual findings” are admissible. Thus the Rule does not specifically
distinguish between facts and opinions contained in those reports.
- “Factual findings” includes opinions and conclusions. 401 and 403 protections will
ensure certain information doesn’t come in.
- (1) Conclusions or opinions in public agency investigative reports are admissible under
Rule 803(8) as long as the conclusion or opinion is based on the factual investigation.
- (2) If a party introduces part of a statement into evidence, the opposing party may present
the remaining portions of the statement to ensure that the court has a complete
understanding of the evidence.

HEARSAY EXCEPTION- RESIDUAL


FRE 807
(a) In General. Under the following conditions, a hearsay statement is not excluded by the rule
against hearsay even if the statement is not admissible under a hearsay exception in Rule 803 or
804:
(1) the statement is supported by sufficient guarantees of trustworthiness—after
considering the totality of circumstances under which it was made and evidence, if any,
corroborating the statement; and
(2) it is more probative on the point for which it is offered than any other evidence that
the proponent can obtain through reasonable efforts.
(b) Notice. The statement is admissible only if the proponent gives an adverse party reasonable
notice of the intent to offer the statement—including its substance and the declarant's name—so
that the party has a fair opportunity to meet it. The notice must be provided in writing before the
trial or hearing—or in any form during the trial or hearing of the court, for good cause, excuses a
lack of earlier notice.
Trustworthiness Requirements (other factors)
Whether the
Whether the out- declarant had Whether the Whether other
of-court firsthand declarant ever evidence
statement was knowledge of recanted the corroborates the
made under oath. facts in the statement. statement.
statement.
Whether other Whether the
Whether that
evidence declarant had any
corroborating
undermines or incentive to lie
evidence is
contradicts the when making the
subject to cross-
out-of-court out-of-court
examination.
statement. statement.
DALLAS COUNTY V. COMMERCIAL UNION ASSURANCE CO. (1961) [P. 575]
- Facts: On July 7, 1957, the clock tower of the Dallas County Courthouse collapsed.
Dallas County had insurance for loss to the courthouse caused by fire or lightning. Dallas
County concluded that the collapse of the courthouse was due to a lightning strike
because the collapse debris contained charcoal and charred timbers. Furthermore,
witnesses reported that a bolt of lightning struck the courthouse on July 2, 1957. The
insurers on the courthouse contested Dallas County’s conclusion that lightning had
caused the collapse. The insurers’ engineers concluded that structural weaknesses
attributable to faulty design and construction had caused the collapse. At trial, the
insurers sought to introduce evidence that the charcoal and char found in the debris came
from an earlier fire. The insurers sought to introduce into evidence a newspaper article
which reports that the courthouse tower had suffered a fire on June 9, 1901 when the
courthouse was still under construction. The newspaper article was admitted, and a jury
found for the insurers.
- Issue: When a fact is generally known throughout the community and the fact occurred
so long ago that the testimony of eyewitnesses is not trustworthy or is unavailable, is a
newspaper article admissible to show the fact?
- Holding: Yes. The requirements for admission of hearsay evidence is that the hearsay
evidence (1) be necessary and (2) be trustworthy. The first requirement means that unless
the hearsay evidence is admitted, the facts it brings out may be lost because there is no
other source readily available for the fact. When a fact occurred long ago, hearsay
evidence may be the only evidence of that fact. The second requirement means that the
circumstances are such that a sincere and accurate statement would naturally be uttered,
or that other considerations, such as a fear of punishment would deter one from lying, or
where a false statement was made under conditions of publicity that an error would have
been detected and corrected.
- Outcome: The hearsay evidence concerning the 1901 fire at the Dallas Courthouse meets
this two-part test. Since the fire occurred 58 years ago, there are likely no eyewitnesses
who were older than children currently alive. If there are any, their memories of the
incident are likely dimmed after 58 years. There is realistically no other way to ascertain
the fact of the fire other than the newspaper article. Second, there would have been no
reason for a newspaper to have falsely reported a fire at the courthouse if there was none.
A reporter would have no motive to falsify such a report, and a false report would have
subjected him and the newspaper to embarrassment. The newspaper article is admissible,
not under a particular hearsay exception, but because it is necessary and trustworthy.
- In matters of local interest, when the fact in question is of such a public nature that it
would generally be known throughout the community and when the fact occurred so long
ago that the testimony of eyewitnesses would be less trustworthy than a newspaper
account, the newspaper article concerning that fact is admissible.
CONFRONTATION CLAUSE (6 T H AMENDMENT)
- In all criminal prosecutions, the accused shall enjoy the right . . . to be confronted with
the
witnesses against him . . . .
Exam Analysis (Confrontation Clause)
- The Confrontation Clause only comes into play where admissible under 800s (hearsay
rules)
- Applies only against the accused in (1) criminal cases, and (2) only limits the prosecutor
- If the statement is non-testimonial, there is no violation
- If the statement is testimonial, the defendant must have had the opportunity to CX the
declarant at trial, or
- Prosecution must establish (1) unavailability (similar standard as under 804(a)) and (2)
the defendant had a prior opportunity to examine the declarant
o There must have been a “similar motive” in the prior CX of the declarant that
there would be at the current trial
HEARSAY AND THE CONFRONTATION CLAUSE
OHIO V. ROBERTS (1980)
- Before Crawford, SCOTUS said 6A was concerned with reliability of hearsay statements,
and thus hearsay was admissible so long as it bore “adequate indica of reliability”
CRAWFORD V. WASHINGTON (2004) [P. 594]
- Facts: Crawford was charged with assault and attempted murder after stabbing a man
who allegedly tried to rape his wife, Sylvia. At trial, the prosecution introduced into
evidence a recorded statement by Sylvia describing the stabbing to police. Sylvia was
unavailable to testify at trial because of the state marital privilege. The jury convicted
Crawford.
- Issue: Is a recorded statement to the police by an unavailable witness admissible at trial?
- Holding: No. The Confrontation Clause of the Sixth Amendment provides that a
defendant has the right to confront the witnesses providing “testimonial” statements
against him with a reasonable opportunity for cross examination. Testimonial statements
consist of in-court testimony and its functional equivalent—statements that the declarant
would reasonably expect to be used by the prosecution. Testimonial statements of
unavailable witnesses are admissible only where the defendant had a prior opportunity for
cross examination.
- Testimonial statements of witnesses not present at trial are admissible only where the
declarant is unavailable and the defendant had a prior opportunity for cross examination.
HAMMON V. INDIANA (2006) [P. 628] AND DAVIS V. WASHINGTON [P. 628]

DAVIS/HAMMON “PRIMARY PURPOSE” TEST


- Statements are non-testimonial when made in the course of police interrogation under
circumstances objectively indicated that the primary purpose of the interrogation is to
enable police assistance to meet an ongoing emergency.
- They are testimonial when the circumstances objectively indicate that there is no such
ongoing emergency, and that the primary purpose of the interrogation is to establish or
prove past events potentially relevant to later prosecution.

Helpful But Non-Exhaustive Factors


Ongoing Emergency Gathering Criminal Evidence
(non-testimonial)(Davis Scenario) (testimonial) (Crawford Scenario)
- Statements describe an ongoing - Statements describe past criminal acts.
incident (presently occurring).  - Statements are about determining who
- Statements were necessary for the is the assailant. 
police to assist the declarant in dealing - Declarant not facing danger.
with the ongoing emergency.  - Declarant is calm. 
- Statements were necessary to protect - Declarant knows statements could be
the police officers (e.g. whether used at trial. 
assailant has a weapon). 
- Declarant is frantic or scared or
emotional. 
MICHIGAN V. BRYANT (2011) [P. 627]
- Expanded the definition of non-testimonial statements
- Indicated that there may be other circumstances, aside from ongoing emergencies, when a
statement in response to police question is non-testimonial

OHIO V. CLARK () [DOWNLOADED FROM CANVAS]


- Are statements made to a teacher (mandatory reporter) testimonial?
- No, and therefore 6A challenge fails. Child’s intent in answering teacher’s questions were
informal/nonconfrontational. Teachers would want to protect children regardless of duty
to report (in their nature) so duty does not make the convo investigatory or the child’s
statement testimonial
- Primary purpose test applies to hearsay by children in response to questions by teachers.
Likely to be found non-testimonial. Looks at intent of declarant and person questioning.
- Out-of-court statements made to persons other than law-enforcement officers are not
excluded from admission into evidence by the Confrontation Clause.
AVOIDING A CONFRONTATION CLAUSE PROBLEM

WHEN CAN WE RULE OUT A CONFRONTATION CLAUSE ISSUE


- The statement is offered in a civil case or against the government in a criminal case 
- The statement is not offered for its truth 
- The statement is hearsay but does not meet a hearsay exception or exemption
- If it is not admissible as a matter of evidence law, it cannot come in – it’s “testimonial-
ity” is irrelevant
- The statement is attributable to the defendant under FRE 801(d)(2)
- The declarant is subject to cross-examination at trial
- The declarant is unavailable, and defendant had a prior opportunity and similar motive to
cross-examine the declarant 
- The defendant forfeited his Confrontation Clause right
- Same standard as FRE 804(6) – require wrongdoing by D that is intended to and does
cause unavailability of declarant as a witness.
Clearly Non-Testimonial Clearly Testimonial
- Casual remark to acquaintance - Solemn declaration made for purpose
- Off-hand, overheard remark of proving some fact
- Co-conspirators’ statements - Formalized testimonial materials (e.g.
- Business records affidavit, prior testimony)
- Party’s own words

STATEMENTS THAT NEVER OR USUALLY DO NOT RAISE A CONFRONTATION


CLAUSE ISSUE
- Statements that almost never raise a confrontation issue:
o FRE 801(d): Prior statement exception/opposing party’s statement
o FRE 803(5) Recorded Recollection 
o FRE 803(6) Business Records
o FRE 803(10) Absence of a Public Record
o FRE 804(b)(1) Former Testimony
o FRE 804(b)(2)  Dying Declaration
o FRE 804(b)(6) Forfeiture by Wrongdoing
o FRE 803(4) Statement Made for Medical Diagnosis or Treatment
o FRE 803(7) Absence of a Record of a Regularly Conducted Activity 
BRUTON V. US (1968) [P. 696]
- Facts: Bruton and Evans were charged with committing armed postal robbery. Bruton
and Evans had a joint trial. At trial, a postal inspector testified that Evans orally
confessed to him that Evans and Bruton had committed the robbery. The confession was
admitted into evidence against Evans, but the trial court instructed the jury that Evans’
confession was inadmissible hearsay against Bruton. Bruton and Evans were both
convicted.
- Issue: Under the Sixth Amendment, may a defendant’s confession incriminating a co-
defendant be admitted if the jury is instructed to disregard the reference to the co-
defendant?
- Holding: No. Admitting one defendant’s confession implicating a co-defendant is a
violation of the Sixth Amendment Confrontation Clause. A jury instruction to disregard
the reference to the co-defendant in assessing the co-defendant’s guilt or innocence is not
a sufficient replacement for the right to cross-examination. If a confessing co-defendant’s
statement directly implicates the non-confessing co-defendant, the confrontation clause is
violated where the statement is admitted in a joint-trial when the confessing co-defendant
is not testifying.
- The admission of a defendant’s confession incriminating a co-defendant violates the
Sixth Amendment Confrontation Clause.

GRAY V. MARYLAND (1998) [P. 709]


- Facts: Bell confessed to the police that he and Kevin Gray had beaten a man to death.
The United States charged Bell and Gray with murder. The district court tried Bell and
Grey jointly and admitted Bell’s confession into evidence. However, the judge ordered
that Gray’s name be redacted. Thus, when the police officer read Bell’s statement into
evidence, he replaced Gray’s name with the word “deleted” or “deletion.” After the
officer read the confession, the prosecutor asked him whether the information he received
from Bell allowed him to then go and arrest Gray. The officer answered in the
affirmative. The prosecution also admitted a written copy of the confession into evidence.
Gray’s name was again redacted. In its place was a blank space separated by commas.
The judge instructed the jury that the confession was evidence against Bell, not Gray. The
jury convicted both men.
- Issue: Are out-of-court statements made by a codefendant that incriminate another
defendant inadmissible at trial even with a limiting instruction or if the name of the
defendant is redacted?
- Holding: Out-of-court statements made by a codefendant that incriminate another
defendant are inadmissible at trial even with a limiting instruction or if the name of the
defendant is redacted. Bruton held that if a codefendant confesses and names another
defendant, the confession is inadmissible at trial, even with a limiting instruction. A jury
often realizes that a confession that replaces a defendant’s name with an obvious blank
space or the word “deleted” refers to the defendant. This may be especially true if a judge
specifically tells the jury not to consider the confession as evidence against the defendant.
Additionally, an obvious deletion can call the jury's attention to the removed name and
encourage the jury to speculate as to who the removed name might be. In this case,
Grey’s name was either replaced with an obvious blank space or the word “deleted” in
the redacted confession. The redacted confession serves as a directly accusatory
statement against Gray.
- Robinson: no Bruton problem with redaction and no mention of non-confessing co-
defendant unless other circumstances make clear it was referencing the confessing co-
defendant
- Out-of-court statements made by a codefendant that incriminate another defendant are
inadmissible at trial even with a limiting instruction or if the name of the defendant is
redacted.
LAY OPINION V EXPERT TESTIMONY
LAY WITNESS OPINION
FRE 701 Lay Opinions
Lay witness opinion must be:
- Based on perception
- Helpful
o Jurors could not have judged the matter for themselves
o Witness’ opinion adds information beyond what is conveyed in “just the facts”
- NOT based on scientific, technical, or other specialized knowledge
Advisory Committee Notes
- Identifies the “prototypical example[s] of the type of evidence contemplated by the
adoption of Rule 701” as “relat[ing] to the appearance of persons or things, identity,
the manner of conduct, competency of a person, degrees of light or darkness, sound, size,
weight, distance, and an endless number of items that cannot be described factually in
words apart from inferences.”
- A lay witness may “testify that a substance appeared to be a narcotic, so long as a
foundation of familiarity with the substance is established…Such testimony is not based
on specialized knowledge within the scope of Rule 702, but rather is based upon a
layperson’s personal knowledge.” If, however, that witness were to describe how a
narcotic was manufactured, or to describe the intricate workings of a narcotic
distribution network, then the witness would have to qualify as an expert under
Rule 702.” 
EXPERT TESTIMONY
FRE 702 Testimony by Expert Witnesses
A witness who is qualified as an expert by knowledge, skill, experience, training, or education
may testify in the form of an opinion or otherwise if:
(a) the expert’s scientific, technical, or other specialized knowledge will help the trier
of fact to understand the evidence or to determine a fact in issue;
(b) the testimony is based on sufficient facts or data;
(c) the testimony is the product of reliable principles and methods; and
(d) the expert has reliably applied the principles and methods to the facts of the case.
Demands on Expert Testimony
o Proper qualifications
o Knowledge, skill, experience, training, or education
o Proper topic
o Standard: Testimony must “help the trier of fact to understand the evidence or to
determine a fact in issue.”
 Two guiding principles:
 Topic must be beyond the knowledge of the average juror
o Improper Topics
 Matters of common knowledge
 Opinions on law FRE 704
 Opinions on credibility (Batangan)
 Opinions on eyewitness identification
 Expert may not tell the jury what verdict to reach
o Sufficient basis
o FRE 602 – need for personal knowledge does not apply to a witness’s expert
testimony. So, what is a proper basis for an expert opinion?
 Three permissible sources for an expert opinion:
 Facts or data she personally observed
 Facts or data made available at trial
o Hypothetical questions
o Testimony of other witnesses (FRE 615)
 Facts or data made aware of before trial
 FRE 705 – disclosing facts & data underlying an expert opinion
o Relevant and reliable methods
o Frye and Daubert
 Daubert Factors:
 Whether theory or technique can be and has been tested
 Whether it has been subjected to peer review and publication
 What are the known or potential error rates
 Existence of standards and controls
 General acceptance (Frye)
US v. Johnson (1979) [p. 750]
- Facts: Six defendants were charged with dealing in marijuana that had been imported
from outside the US. The prosecution called John de Pianelli to testify as an expert
witness that the marijuana dealt by defendants came from Colombia. De Pianelli
allegedly helped the defendants’ smuggling ring by unloading it and smoking some of the
marijuana each time to make sure it was Colombian. De Pianelli said he had smoked
marijuana over 1,000 times and that he had been asked to identify marijuana over 100
times and had never made a mistake. He based his identification on the marijuana’s
appearance, smell and the effect of smoking it. The defendants objected to de Pianelli as
an expert witness. The district court permitted de Pianelli’s testimony and the defendants
were convicted.
- Issue: Is a witness qualified to be an expert witness when the witness’s expertise stems
from experience, not from formal training or education?
- Holding: Yes. FRE 702 provides that expertise may be obtained from experience as well
as formal training. Expert testimony is warranted when (1) the subject of the testimony is
distinctly related to some science, profession, business or occupation as to be beyond the
knowledge of the average layman; and (2) the witness has such knowledge or experience
in that field or calling as to make it appear that his opinion or inference will probably aid
the trier in his search for truth.
- A witness is qualified to be an expert witness when the area of expertise is beyond the
knowledge of the average layman and the witness’s expertise stems from experience.

IMPROPER TOPICS OF EXPERT TESTIMONY


HYGH V. JACOBS (1992) [P. 760]
- Facts: Officer Jacobs struck Hygh on the cheek. The blow broke Hygh’s cheekbones, and
plastic surgery was needed to repair the injury. Hygh claimed that Jacobs struck him
while Hygh was bending down to pick up his jacket after Jacobs told him he was under
arrest. Jacobs claimed that he struck Hygh in self-defense. Hygh sued Jacobs for alleged
constitutional violations related to his arrest and the blow by Jacobs. Hygh called Terry
Cox, an expert witness concerning law enforcement. Cox testified that Jacobs had used
“deadly physical force” that was not “warranted under the circumstances.” Cox defined
“deadly physical force” as “using force in such a way that it has the potential to kill
someone.”
- Issue: May an expert witness express his opinion as to ultimate legal conclusions?
- Holding: No. Although FRE 704 does not exclude expert testimony that goes to an
ultimate issue, it does exclude expert testimony that addresses an ultimate legal issue.
This prevents against the risk of an expert telling the jury what conclusion to reach. Even
if the jury were not led into following an expert, testimony as to an ultimate legal
conclusion would be objectionable because it tells the jury what standard to adopt. It is
the province of the judge to instruct the jury. In this case, Cox’s testimony was not
proper, because he defined “deadly physical force,” and New York already provides a
statutory definition of deadly physical force. Further, Cox’s testimony that Jacobs's
conduct was not “warranted under the circumstances” was also improper, because this
testimony effectively told the jury what conclusion to reach.
- Outcome: Judgement affirmed, because it was harmless error. There are only minimal
differences between Cox's definition and the statutory definition, and the judge
thoroughly instructed the jury regarding the standards for measuring use of force, and the
jury could have reached the same conclusion as Cox based on this larger body of
testimony.
- An expert witness may not express his opinion as to ultimate legal conclusions.
STATE V. BATANGAN (1990) [P. 766]
- Facts: Batangan was charged with sexual abuse of his daughter. The victim alleged that
Batangan physically and sexually abused her when she was six or seven years old. The
victim could not provide any specific dates nor were any of the acts specific to one
incident or another. There were no witnesses to the alleged acts. The victim did not report
the abuse until several months after they occurred when she reported physical abuse to
school officials. She then recanted the allegations of physical abuse but accused Batangan
of sexual abuse. The victim then recanted the allegations of sexual abuse, but testified to
them at trial. Batangan was tried on charges of sexual abuse. At trial, the prosecution
presented Dr. John Bond as an expert in the field of clinical psychology with a
subspecialty in sexually abused children. Bond testified as to how he evaluates whether a
child is telling the truth about sex abuse. Bond then implicitly testified that the victim
here was believable and that he believed she had been abused by Batangan. Batangan was
convicted of first-degree sex abuse.
- Issue: In sex abuse cases, may an expert testify as to whether he believes the victim’s
story about being abused?
- Holding: No. Expert testimony with its aura of special reliability and trustworthiness has
the danger of unduly influencing the jury. According to Hawaii Rule of Evidence 702
(similar to Federal Rule of Evidence 702), expert testimony is only permitted in
knowledge not possessed by the average layman who does not have the expert’s special
training or skill. However, the average layman has the capacity to judge a witness’s
credibility. Thus the credibility of a witness is not a proper area for expert testimony.
However, sexual abuse of children is a relatively unknown phenomenon and an average
layman may not be able to assess the credibility of a child who complains of sexual
abuse. Child victims have exhibited unique behaviors which may seem inconsistent with
the norms of other victims of assault, such as delayed reporting of the assault and
recanting the assault. It is proper for an expert to testify as to the unique behaviors
exhibited by child victims of sex abuse, but it is improper for the expert to testify, as
Bond did here, as to his conclusion that abuse occurred here. Once the jury has learned
that the victim’s behavior is consistent with that of other child victims of sex abuse, it can
draw its own conclusions.
- In sex abuse cases, an expert may not testify as to whether he believes the victim’s story
about being abused.
STATE V GUILBERT P 771
- Facts: A man was shot in a bar, police observed Guilbert running away from the bar,
Williams and Ross (two potential witnesses) were later found shot dead in a car that was
crashed into a tree. Nine days later, Baldwin saw Guilbert’s photo and told police she saw
him running from Ross and Williams car. Her cousin Gomez corrobated her testimony at
trial. Lang, testified that Guilbert also was the person who shotthe man in the bar.
Guilbert attempted to introduce an expert testimony of a doctor who would have testified
as to how stress affects memory and it’s accuracy.
- Issue: Is expert testimony on the fallibility of eyewitness identification admissible when
that testimony would assist the jury in evaluating the identification?
- Holding: Yes. This court finds that Kemp and McClendon must be overruled, as it is
commonly accepted among the scientific community and the courts that the average juror
is unaware of factors affecting the reliability of eyewitness testimony. Because Baldwin
and Gomez knew Guilbert personally, the court ruled these witnesses were reliable
enough to not need the expert testimony. Lang, however did not know Guilbert outside of
this incident and therefore the court found that the trial court abused its discretion by
precluding the expert testimony in regard to Lang’s identification.
- Expert testimony on the fallibility of eyewitness identification is admissible when that
testimony would assist the jury in evaluating the identification.

BASIS OF EXPERT TESTIMONY


FRE 703
Experts can rely on facts or data that might not be admissible at trial (i.e., hearsay) if it’s the facts
or data normally relied upon by other experts in the field.

Three permissible sources for an expert opinion:


- Facts or data she personally observed
- Facts or data made available at trial
- Hypothetical questions
- Testimony of other witnesses (expert may remain in courtroom – FRE 615)
- Facts or data made aware of before trial

FRYE V. US (1923) [P. 793]


- Facts: Frye was charged with second-degree murder. At trial, Frye’s counsel sought to
introduce an expert who would testify as to a “systolic blood pressure deception test” that
the expert performed on Frye. The test would allegedly show whether Frye was lying
when he testified because blood pressure allegedly rises when a person lies but stays the
same when the person told the truth. The trial court rejected the testimony of the expert
because it had not gained recognition/acceptance in the field.
- Issue: May expert scientific evidence be admitted if the science has gained general
acceptance in the relevant scientific community?
- Holding: Yes. The standard for admission of expert scientific evidence is that evidence is
admissible if the science has gained general acceptance in the relevant scientific
community.
- Expert scientific evidence is only admissible if the science has gained general acceptance
in the relevant scientific community.
DAUBERT V. MERRELL DOW PHARMACEUTICALS (1993) [P. 794]
- Facts: Daubert and Schuller were minors born with birth defects. The minors and their
parents brought suit against Merrell Dow Pharmaceuticals alleging that its product,
Bendectin, caused the defects. Daubert brought forth the testimony of eight scientific
experts who had concluded that Bendectin could cause birth defects. However, the
district court granted Merrell’s motion for summary judgment and the United States
Court of Appeals for the Ninth Circuit affirmed because Daubert’s experts’ opinions were
based on scientific techniques that were not “generally accepted” in the scientific
community and as a result, their testimony was not admissible under Frye.
- Issue: Must an expert witness’s scientific knowledge be “generally accepted” in the
relevant field to be admissible?
- Holding: No. Frye held that an expert witness’s scientific knowledge must be “generally
accepted” in the relevant field to be admissible. However, the Court overrules Frye
because since-enacted Rule 702 and its legislative history do not mention Frye or the
“generally accepted” test. Rule 702 provides that expert testimony is admissible if it will
assist the jury in comprehending the evidence and determining issues of fact.
- Reasoning: To determine whether scientific knowledge is admissible as expert
testimony, the court must make a preliminary determination under Rule 104(a) that the
reasoning behind the testimony is scientifically valid and can be applied to the facts of
the case. Such a determination should be made by taking into account a number of
factors, including, but not limited to the following: the testability of the
theory/methodology; whether the theory has been published and subject to peer review;
any potential rate of error; and finally, whether the knowledge has reached general
acceptance (the test laid out in Frye).

STEPS FOR ANALYZING IF THERE IS A PRIVILEGE


Who holds the privilege?
When does the privilege apply?
What does the privilege include?
How strong is the privilege?

- Expert testimony from a qualified expert is admissible if it will assist the jury in
comprehending the evidence and determining issues of fact.
PRIVILEGES
Why do we have privileges?
- We want to exclude relevant and probative evidence that will protect a socially beneficial
relationship and/or social right/norm.
Who determines if there is a privilege?
- The judge – 104(a)
Can the judge hear the privileged information in determining whether a privilege exists?
- No, FRE 1101(c)
ATTORNEY-CLIENT PRIVILEGE
Elements of Attorney Client Privilege
- A client or the client’s representative
- An attorney or their representative
- A communication between those two parties
- Confidentiality of those communications
- A purpose facilitating professional legal services to the client

DAUBERT FACTORS
1. Whether theory or technique can be and has been tested
2. Whether it has been subjected to peer review and
publication
3. What are the known or potential error rates
4. Existence of standards and controls
5. General acceptance (Frye)

INADVERTENT DISCLOSURE ANALYSIS


- Multifactor test (pre-FRE 502) for determining whether inadvertent disclosure is a waiver
- Reasonableness of precautions taken
- Time taken to rectify the error
- The scope of discovery
- The extent of the disclosure
- The overriding issue of fairness
- The number of documents to be reviewed
- The time constraints for the production
WILLIAMS V. DISTRICT OF COLUMBIA (2011) [P. 985]
- Facts: Williams sued D.C. on grounds that the D.C. violated the D.C. Whistleblower
Protection Act. Around June 2, 2008, during the discovery phase of the litigation, the
District produced documents to Williams. The response included a “recommendation to
terminate packet” that consisted in part of an email to which the then-Deputy General
Counsel for the District’s Department of Health was a party. The email discussed
Williams’ proposed termination. The District later realized it had inadvertently produced
the email and, on November 22, 2008, wrote to Williams demanding the immediate
return of the email and forbidding its use or disclosure. Williams never responded to the
demand and the District did not follow up in the next two years and eight months of
litigation. In July 2011, Williams listed the email on her exhibit list. On July 20, 2011,
the District moved to exclude the email.
- Issue: Is the attorney-client privilege waived by the inadvertent disclosure of privileged
information if the holder of the privilege fails to take reasonable steps to prevent the
disclosure and fails to take reasonable steps to remedy the error?
- Holding: Under FRE 502(b), the inadvertent disclosure of a document covered by the
attorney-client privilege does not constitute a waiver of the privilege if: (1) the disclosure
was inadvertent; (2) the holder of the privilege took reasonable steps to prevent the
disclosure of the privileged document; and (3) the holder of the privilege promptly took
reasonable steps to remedy the disclosure. If these elements are not satisfied, the privilege
is waived.
- Outcome: The District has not shown it took reasonable steps to prevent inadvertent
disclosure of the email. The District has not explained the methodology it employed to
review and produce documents. It merely makes a conclusory statement that a review
was conducted. The District has also failed to indicate the size of the production as a
whole, which makes it impossible for this court to determine whether the inadvertent
disclosure was reasonable in light of the size of the production. Moreover, the District
provides no indication of how burdensome the document demand was, or what kind of
time pressure the District was under. Consequently, this court finds that the District did
not take reasonable steps to prevent disclosure. The District has also failed to demonstrate
that it took reasonable steps to remedy the disclosure. The District did notify Williams
about the inadvertent disclosure. However, when Williams failed to respond, the District
was required to take further action. Instead, the District failed to resolve the dispute with
Williams and waited two years and eight months before it brought the issue before this
court.
- The attorney-client privilege is waived by the inadvertent disclosure of privileged
information if the holder of the privilege fails to take reasonable steps to prevent the
disclosure and fails to take reasonable steps to remedy the error.

CRIME FRAUD EXCEPTION TO ATTORNEY-CLIENT PRIVILEGE


Conditions for Exception to Apply
- Client seeking to commit or intending to commit a fraud or a crime
- Communication with lawyer is aimed at furthering that crime
TWO MARITAL PRIVILEGES

SPOUSAL TESTIMONIAL PRIVILEGE


Spousal testimonial privilege
- Two requirements to invoke privilege
o currently married; and
o a communication or action
- Arises when a spouse is a criminal defendant (or when before GJ)
- Includes all statements, actions, and observations from before and during the marriage as
long as married when privilege is invoked.
- Rationale: marital relationships are good and valuable and want to encourage the
relationship
TRAMMEL V. US (1980) [P. 1044]
o RULE: One spouse (the criminal defendant) cannot prevent another spouse from
testifying if they want to. The testifying spouse holds the privilege.
o Facts: Heroin importation conspiracy. Hubs goes to trial. Wife offered leniency in
exchange for testimony. Husband argues that he must waive the spousal testimonial
privilege before in order for her to be able to testify against him.
o Hawkins – Rule pre-Trammel said that both spouses had to waive this privilege
o Issue: Should the federal common-law spousal testimonial privilege be modified to allow
a spouse to voluntarily testify against the other?
o Holding: Yes, testifying spouse holds the privilege.
o Reasoning: A rule of evidence that permits an accused to prevent adverse spousal
testimony seems far more likely to frustrate justice than to foster family peace. “It hardly
seems conducive to the preservation of the marital relation to place a wife in jeopardy
solely by…her husband’s control over her testimony.”

MARITAL CONFIDENCES PRIVILEGE


Marital confidence privilege
o Four requirements to invoke privilege:
o Married (at one point, don’t have to currently be married)
o Communication
o No third-party present (confidential communication)
o Communication was during course of marriage
o Privilege relates solely to confidential communication, not actions.
o Privilege survives the marriage
o Both spouses control; criminal defendant can block (ex)spouse from testifying against
them
FEDERAL RULES OF EVIDENCE
RUle 101
(a) Scope. These rules apply to proceedings in United States courts. The specific courts and
proceedings to which the rules apply, along with exceptions, are set out in Rule 1101.
(b) Definitions. In these rules:
(1) “civil case” means a civil action or proceeding;
(2) “criminal case” includes a criminal proceeding;
(3) “public office” includes a public agency;
(4) “record” includes a memorandum, report, or data compilation;
(5) a “rule prescribed by the Supreme Court” means a rule adopted by the Supreme Court
under statutory authority; and
(6) a reference to any kind of written material or any other medium includes electronically
stored information.
Rule 102 Purpose
These rules should be construed so as to administer every proceeding fairly, eliminate
unjustifiable expense and delay, and promote the development of evidence law, to the end of
ascertaining the truth and securing a just determination.
Rule 103. Rulings on evidence
a) Preserving a Claim of Error. A party may claim error in a ruling to admit or exclude
evidence only if the error affects a substantial right of the party and:
1. if the ruling admits evidence, a party, on the record:
A) timely objects or moves to strike; and
B) states the specific ground, unless it was apparent from the context; or
2. if the ruling excludes evidence, a party informs the court of its substance by an
offer of proof, unless the substance was apparent from the context.
b) Not Needing to Renew an Objection or Offer of Proof. Once the court rules
definitively on the record — either before or at trial — a party need not renew an
objection or offer of proof to preserve a claim of error for appeal.
c) […]
d) Preventing the Jury from Hearing Inadmissible Evidence. To the extent practicable,
the court must conduct a jury trial so that inadmissible evidence is not suggested to the
jury by any means.
e) Taking Notice of Plain Error. A court may take notice of a plain error affecting a
substantial right, even if the claim of error was not properly preserved.
Rule 104. Preliminary Questions
a) In general. The court must decide any preliminary question about whether a witness
is qualified, a privilege exists, or evidence is admissible. In so deciding, the court is not
bound by evidence rules, except those on privilege.
b) Relevance That Depends on Fact (Conditional)
- When the relevance of evidence depends on whether a fact exists, proof must be
introduced sufficient to support a finding that the fact does exist. The court may admit the
proposed evidence on the condition that the proof be introduced later.
o Advisory Committee Notes
o Subdivision (b).
 In some situations, the relevancy of an item of evidence, in the large sense,
depends upon the existence of a particular preliminary fact. […]
Relevance in this sense has been labelled “conditional relevancy.”
Morgan, Basic Problems of Evidence 45–46 (1962).
c) Conducting a Hearing So That the Jury Cannot Heat It the court must conduct any hearing
on a preliminary question so that the jury cannot hear it if:
1. The hearing involves the admissibility of a confession;
2. A defendant in a criminal case as a witness and so request; or
3. Justice so requires
d) Cross- Examining a Defendant in a Criminal Case. By testifying on a preliminary question,
a defendant in a criminal case does not become subject to cross-examination on other issues in
the case.
e) Evidence Relevant to Weight and Credibility. This role does not limit the parties right to
energies before the jury evidence that is relevant to the way or credibility of evidence
Rule 105
- If the court admits evidence that is admissible against a party or for a purpose — but not
against another party or for another purpose — the court, on timely request, must restrict
the evidence to its proper scope and instruct the jury accordingly.

FRE 106
If a party introduces all or part of a writing or recorded statement, an adverse party may require
the introduction, at that time, of any other part — or any other writing or recorded statement —
that in fairness ought to be considered at the same time.
Rule 401 Test for relevant evidence
• Evidence is relevant if:
o (a) it has any tendency to make a fact more or less probable than it would be
without the evidence
 Key Question: Does the evidence have any tendency to make the
material fact more likely?
o (b) the fact is of consequence in determining the action.
 Key Question:
 What is the substantive law governing the dispute?
 What is the party’s theory of the case?
Rule 402 General Admissibility of Relevant Evidence
• Relevant evidence is admissible unless any of the following provides otherwise: the
United States Constitution; a federal statute; these rules; or other rules prescribed by the
Supreme Court. Irrelevant evidence is not admissible.
Rule 403 Excluding Relevant Evidence
• The court may exclude relevant evidence if:
o its probative value is substantially outweighed 
o By one or more of the following:
 unfair prejudice,
 confusing the issues,
 misleading the jury,
 undue delay,
 wasting time,
 or needlessly presenting cumulative evidence.
FRE 404 Character Evidence; Crimes or other
• (a) Character Evidence
o (1) Prohibited Uses. Evidence of a person’s character or character trait is not
admissible to prove that on a particular occasion the person acted in accordance
with the character or trait.
o (2) Exceptions for a Defendant or Victim in a Criminal Case
 (A) a defendant may offer evidence of the defendant’s pertinent trait, and
if the evidence is admitted, the prosecutor may offer evidence to rebut it;
 (B) subject to the limitations in Rule 412, a defendant may offer evidence
of an alleged victim’s pertinent trait, and if the evidence is admitted, the
prosecutor may:
(i) offer evidence to rebut it; and
(ii) offer evidence of the defendant’s same trait; and
 (C) in a homicide case, the prosecutor may offer evidence of the alleged
victim’s trait of peacefulness to rebut evidence that the victim was the first
aggressor.
o (3) Exceptions for a Witness
Evidence of a witness’s character may be admitted under Rule 607, 608, and 609
• (b) Other Crimes, Wrongs, or Acts
o (1) Prohibited Uses.  Evidence of a crime, wrong, or other act is not admissible to
prove a person’s character in order to show that on a particular occasion the
person acted in accordance with the character.  
o (2) Permitted Uses. Notice in a Criminal Case
 This evidence may be admissible for another purpose, such as proving
motive, opportunity, intent, preparation, plan, knowledge, identity,
absence of mistake, or lack of accident.
o (3) Notice in a Criminal Case.
 In a criminal case, the prosecutor must:
 A. provide reasonable notice of any such evidence that the prosecutor
intends to offer at trial, so that the defendant has a fair opportunity to meet
it;
 B. articulate in the notice the permitted purpose for which the prosecutor
intends to off the evidence and the reasoning that supports the purpose;
and
 C. do so in writing before the rial or in any form during trial if the court,
for good cause, excuses lack of pretrial notice
FRE 405 Methods of Proving Character
(a) By Reputation or Opinion. When evidence of a person’s character or character trait is
admissible, it may be proved by testimony about the person’s reputation or by testimony in the
form of an opinion. On cross-examination of the character witness, the court may allow an
inquiry into relevant specific instances of the person’s conduct.
(b) By Specific Instances of Conduct. When a person’s character or character trait is an
essential element of a charge, claim, or defense, the character or trait may also be proved by
relevant specific instances of the person’s conduct.
- entrapment
- Liable slander
- Child custody
FRE 406 Habit; Routine Practice
- Evidence of a person’s habit or an organization’s routine practice may be admitted to
prove that on a particular occasion the person or organization acted in accordance with
the habit or routine practice. The court may admit this evidence regardless of whether it
is corroborated or whether there was an eyewitness.
FRE 407 Subsequent remedial measure
When measures are taken that would have made an earlier injury or harm less likely to occur,
evidence of the subsequent measures is not admissible to prove:
- negligence;
- culpable conduct;
- a defect in a product or its design; or
- a need for a warning or instruction.
But the court may admit this evidence for another purpose, such as impeachment or — if
disputed — proving ownership, control, or the feasibility of precautionary measures.
FRE 408 compromise offers and negotiation
• (a) Prohibited Uses. Evidence of the following is not admissible — on behalf of
any party — either to prove or disprove the validity or amount of a disputed claim
or to impeach by a prior inconsistent statement or a contradiction:
• (1) furnishing, promising, or offering — or accepting, promising to accept,
or offering to accept — a valuable consideration in compromising or
attempting to compromise the claim; and
• (2) conduct or a statement made during compromise negotiations about
the claim — except when offered in a criminal case and when the
negotiations related to a claim by a public office in the exercise of its
regulatory, investigative, or enforcement authority.
• (b) Exceptions:
• The court may admit this evidence for another purpose, such as proving a
witness’s bias or prejudice, negating a contention of undue delay, or
proving an effort to obstruct a criminal investigation or prosecution.
FRE 409- Payment of Medical Expenses
- Evidence of furnishing, promising to pay, or offering to pay medical, hospital, or similar
expenses resulting from an injury is not admissible to prove liability for the injury.
FRE 410 pleas, plea discussions, and related statements
(a) Prohibited Uses. In a civil or criminal case, evidence of the following is not
admissible against the defendant who made the plea or participated in the plea
discussions:
(1) a guilty plea that was later withdrawn;
(2) a nolo contendere plea;
(3) a statement made during a proceeding on either of those pleas under Federal Rule of
Criminal Procedure 11 or a comparable state procedure; or
(4) a statement made during plea discussions with an attorney for the prosecuting
aturhoity if the discussions did not result in a guilty plea or they resulted in a later-
withdrawn guilty plea

(b) Exceptions. The court may admit a statement described in Rule 410(a)(3) or (4):
(1) in any proceeding in which another statement made during the same play or
plea discussions has been introduced, if in fairness the statements are to be considered
together; or
(2) in a criminal proceeding for perjury or false statement, if the defendant made
the statement under oath, on the record, and with counsel present.
FRE 411- Liability Insurance NOT HEALTH INSURANCE
- Evidence that a person was or was not insured against liability is not admissible to
prove whether the person acted negligently or otherwise wrongfully. But the court
may admit this evidence for another purpose, such as proving a witness’s bias or
prejudice or proving agency, ownership, or control.
FRE 502
The following provisions apply, in the circumstances set out, to disclosure of a communication
or information covered by the attorney-client privilege or work-product protection.
(a) Disclosure Made in a Federal Proceeding or to a Federal Office or Agency; Scope of a
Waiver. When the disclosure is made in a federal proceeding or to a federal office or agency and
waives the attorney-client privilege or work-product protection, the waiver extends to an
undisclosed communication or information in a federal or state proceeding only if:
(1) the waiver is intentional;
(2) the disclosed and undisclosed communications or information concern the same
subject matter; and
(3) they ought in fairness to be considered together.
(b) Inadvertent Disclosure. When made in a federal proceeding or to a federal office or agency,
the disclosure does not operate as a waiver in a federal or state proceeding if:
(1) the disclosure is inadvertent;
(2) the holder of the privilege or protection took reasonable steps to prevent disclosure;
and
(3) the holder promptly took reasonable steps to rectify the error, including (if applicable)
following Federal Rule of Civil Procedure 26 (b)(5)(B).
(c) Disclosure Made in a State Proceeding. When the disclosure is made in a state proceeding
and is not the subject of a state-court order concerning waiver, the disclosure does not operate as
a waiver in a federal proceeding if the disclosure:
(1) would not be a waiver under this rule if it had been made in a federal proceeding; or
(2) is not a waiver under the law of the state where the disclosure occurred.
(d) Controlling Effect of a Court Order. A federal court may order that the privilege or
protection is not waived by disclosure connected with the litigation pending before the court —
in which event the disclosure is also not a waiver in any other federal or state proceeding.
(e) Controlling Effect of a Party Agreement. An agreement on the effect of disclosure in a
federal proceeding is binding only on the parties to the agreement, unless it is incorporated into a
court order.
(f) Controlling Effect of this Rule. Notwithstanding Rules 101 and 1101, this rule applies to
state proceedings and to federal court-annexed and federal court-mandated arbitration
proceedings, in the circumstances set out in the rule. And notwithstanding Rule 501, this rule
applies even if state law provides the rule of decision.
(g) Definitions. In this rule:
(1) “attorney-client privilege” means the protection that applicable law provides for
confidential attorney-client communications; and
(2) “work-product protection” means the protection that applicable law provides for
tangible material (or its intangible equivalent) prepared in anticipation of litigation or for
trial.
FRE 601 Competency to Testify in General
- Every person is competent to be a witness unless these rules provide otherwise. But in a
civil case, state law governs the witness’s competency regarding a claim or defense for
which state law supplies the rule of decision.
FRE 602 Need for Personal Knowledge
- A witness may testify to a matter only if evidence is introduced sufficient to support a
finding that the witness has personal knowledge of the matter. Evidence to prove personal
knowledge may consist of the witnesses and testimony. This rule does not apply to
witnesses expert testimony and a rule 703.
FRE 603
- Before testifying, a witness must give an oath or affirmation to testify truthfully. It must
be in a form designed to impress that duty on the witness’s conscience.
FRE 604
An interpreter must be qualified and must give an oath or affirmation to make a true translation.
FRE 605
The presiding judge may not testify as a witness at the trial. A party need not object to preserve
the issue.
FRE 606
(a) At the Trial. A juror may not testify as a witness before the other jurors at the trial. If a juror
is called to testify, the court must give a party an opportunity to object outside the jury’s
presence.
(b) During an Inquiry into the Validity of a Verdict or Indictment.
(1) Prohibited Testimony or Other Evidence. During an inquiry into the validity of a verdict or
indictment, a juror may not testify about any statement made or incident that occurred during the
jury’s deliberations; the effect of anything on that juror’s or another juror’s vote; or any juror’s
mental processes concerning the verdict or indictment. The court may not receive a juror’s
affidavit or evidence of a juror’s statement on these matters.
(2) Exceptions. A juror may testify about whether:
(A) extraneous prejudicial information was improperly brought to the jury’s attention;
(B) an outside influence was improperly brought to bear on any juror; or
(C) a mistake was made in entering the verdict on the verdict form.
FRE 607 Any party, including the party that called the witness, may attack the witness’s
credibility
FRE 608 A Witness’s Chracter for Truthfulness or Untruthfulness
(a) Reputation or Opinion Evidence. A witness’s credibility may be attacked or supported by
testimony about the witness’s reputation for having a character for truthfulness or untruthfulness,
or by testimony in the form of an opinion about that character. But evidence of truthful
character is admissible only after the witness’s character for truthfulness has been
attacked.
(b) Specific Instances of Conduct. Except for a criminal conviction under Rule 609, extrinsic
evidence is not admissible to prove specific instances of a witness’s conduct in order to
attack or support the witness’s character for truthfulness. But the court may, on cross-
examination, allow them to be inquired into if they are probative of the character for truthfulness
or untruthfulness of:
(1) the witness; or
(2) another witness whose character the witness being cross-examined has testified
about.* * *
By testifying on another matter, a witness does not waive any privilege against self-incrimination
for testimony that relates only to the witnesses character for truthfulness.
FRE 609
(a) In General. The following rules apply to attacking a witness’s character for truthfulness by
evidence of a criminal conviction:
(1) for a crime that, in the convicting jurisdiction, was punishable by death or by
imprisonment for more than one year, the evidence:
(A) must be admitted, subject to Rule 403, in a civil case or in a criminal case in
which the witness is not a defendant; and
(B) must be admitted in a criminal case in which the witness is a defendant, if the
probative value of the evidence outweighs its prejudicial effect to that defendant; and
(2) for any crime regardless of the punishment, the evidence must be admitted if the court
can readily determine that establishing the elements of the crime required proving — or the
witness’s admitting — a dishonest act or false statement.
(b) Limit on Using the Evidence After 10 Years. This subdivision (b) applies if more than 10
years have passed since the witness’s conviction or release from confinement for it, whichever is
later. Evidence of the conviction is admissible only if:
(1) its probative value, supported by specific facts and circumstances, substantially
outweighs its prejudicial effect; and
(2) the proponent gives an adverse party reasonable written notice of the intent to use it
so that the party has a fair opportunity to contest its use.
(c) Effect of a Pardon, Annulment, or Certificate of Rehabilitation. Evidence of a conviction is
not admissible if:
(1) the conviction has been the subject of a pardon, annulment, certificate of
rehabilitation, or other equivalent procedure based on a finding that the person has been
rehabilitated, and the person has not been convicted of a later crime punishable by death or by
imprisonment for more than one year; or
(2) the conviction has been the subject of a pardon, annulment, or other equivalent
procedure based on a finding of innocence.
(d) Juvenile Adjudications. Evidence of a juvenile adjudication is admissible under this rule only
if:
(1) it is offered in a criminal case;
(2) the adjudication was of a witness other than the defendant;
(3) an adult’s conviction for that offense would be admissible to attack the adult’s
credibility; and
(4) admitting the evidence is necessary to fairly determine guilt or innocence.
(e) Pendency of an Appeal. A conviction that satisfies this rule is admissible even if an appeal is
pending. Evidence of the pendency is also admissible.
FRE 610 Evidence of a witness’s religious beliefs or opinions is not admissible to attack or support the witness’s
credibility.
FRE 611
(a) Control by the Court; Purposes. The court should exercise reasonable control over the mode
and order of examining witnesses and presenting evidence so as to:
(1) make those procedures effective for determining the truth;
(2) avoid wasting time; and
(3) protect witnesses from harassment or undue embarrassment.
(b) Scope of Cross-Examination. Cross-examination should not go beyond the subject matter of
the direct examination and matters affecting the witness’s credibility. The court may allow
inquiry into additional matters as if on direct examination.
(c) Leading Questions. Leading questions should not be used on direct examination except as
necessary to develop the witness’s testimony. Ordinarily, the court should allow leading
questions:
(1) on cross-examination; and
(2) when a party calls a hostile witness, an adverse party, or a witness identified with an
adverse party.
FRE 612 Writing Used to Refresh a Witness
- a) Scope. This rule give an adverse party certain options when a wtiness uses a writing to
refresh memory:
o 1. While testifying; or
o 2. Before testifyinf, if the court decides that justice requires the party to have
those options.
- b) Adverse Party’s Options; Deleting Unrelated Matter. Unless 18 USC Section 3500
provides otherwise in a criminal case, an adverse party is entitled to have the writing
produced at the hearing, to inspect it, to cross-examine the witness about it, and to
introduce an evidence any portion that relates to the witnesses testimony. If the producing
party claims that the writing includes unrelated matter, the court must examine the
writing and camera, delete any unrelated portion, and order that the rest be delivered to
the adverse party. Any portion deleted over objection must be preserved for the record.
- c) Failure to Produce or Deliver the Writing. If the writing is not produced or is not
delivered as ordered, the court may issue an appropriate order. But if the prosecution does
not comply in a criminal case, the court must strike the witnesses testimony or, if justice
so requires, declare a mistrial.
FRE 613
(a) Showing or Disclosing the Statement During Examination. When examining a witness about
the witness’s prior statement, a party need not show it or disclose its contents to the witness. But
the party must, on request, show it or disclose its contents to an adverse party’s attorney.
(b) Extrinsic Evidence of a Prior Inconsistent Statement. Extrinsic evidence of a witness’s prior
inconsistent statement is admissible only if the witness is given an opportunity to explain or deny
the statement and an adverse party is given an opportunity to examine the witness about it, or if
justice so requires. […] 
FRE 614
(a) Calling. The court may call a witness on its own or at a party’s request. Each party is entitled
to cross-examine the witness.
(b) Examining. The court may examine a witness regardless of who calls the witness.
(c) Objections. A party may object to the court’s calling or examining a witness either at that
time or at the next opportunity when the jury is not present.
FRE 615
At a party’s request, the court must order witnesses excluded so that they cannot hear other
witnesses’ testimony. Or the court may do so on its own. But this rule does not authorize
excluding:
(a) a party who is a natural person;
(b) an officer or employee of a party that is not a natural person, after being designated as the
party’s representative by its attorney;
(c) a person whose presence a party shows to be essential to presenting the party’s claim or
defense; or
(d) a person authorized by statute to be present.
FRE 701
If a witness is not testifying as an expert, testimony in the form of an opinion is limited to one
that is:
a) rationally based on the witness’s perception
b) helpful to clearly understanding the witness’s testimony or to determining a fact in issue
c) not based on scientific, technical, or other specialized knowledge within the scope or Rule 702
FRE 702
A witness who is qualified as an expert by knowledge, skill, experience, training, or
education may testify in the form of an opinion or otherwise if:
(a) the expert’s scientific, technical, or other specialized knowledge will help the trier
of fact to understand the evidence or to determine a fact in issue;
(b) the testimony is based on sufficient facts or data;
(c) the testimony is the product of reliable principles and methods; and
(d) the expert has reliably applied the principles and methods to the facts of the case.
FRE 703
Experts can rely on facts or data that might not be admissible at trial (i.e., hearsay) if it’s the facts
or data normally relied upon by other experts in the field. If experts in the particular field would
reasonably rely on those kinds of facts or data in forming an opinion on the subject, they need not be admissible for
the opinion to be admitted. But if the facts or data would otherwise be inadmissible, the proponent of the opinion
may disclose them to the jury only if their probative value in helping the jury evaluate the opinion substantially
outweighs their prejudicial effect.
FRE 704
(a) In General — Not Automatically Objectionable. An opinion is not objectionable
just because it embraces an ultimate issue.
(b) Exception.  In a criminal case, an expert witness must not state an opinion about
whether the defendant did or did not have a mental state or condition that constitutes an element
of the crime charged or of a defense.  Those matters are for the trier of fact alone.
FRE 705
Unless the court orders otherwise, an expert may state an opinion — and give the reasons for it
— without first testifying to the underlying facts or data. But the expert may be required to
disclose those facts or data on cross-examination.
FRE 801
(a) Statement. “Statement” means a person’s oral assertion, written assertion, or nonverbal
conduct, if the person intended it as an assertion.
(b) Declarant. Means the person who made the statement
(c) Hearsay. Means a statement that
1. the declarant does not make while testifying at the current trial or hearing; and
2. a party offers in evidence to prove the truth of the matter asserted in the statement
(d) Statements That Are Not Hearsay. A statement that meets the following conditions is not
hearsay:
***
(1) A Declarant-Witness’s Prior Statement. The declarant testifies and is subject to
cross-examination about a prior statement, and the statement:
(A) is inconsistent with the declarant’s testimony and was given under penalty of
perjury at a trial, hearing, or other proceeding or in a deposition;
(B) is consistent with the declarant’s testimony and is offered:
(i) to rebut an express or implied charge that the declarant recently
fabricated it or acted from a recent improper influence or motive in so
testifying; or
(ii) to rehabilitate the declarant's credibility as a witness when attacked on
another ground; or
(C) identifies a person as someone the declarant perceived earlier.
(2) An Opposing Party’s Statement. The statement is offered against an opposing party
and:
A) was made by the party in an individual or representative capacity
(B) is one the party manifested that it adopted or believed to be true;
(C) was made by a person whom the party authorized to make a statement on the
subject;
(D) was made by the party’s agent or employee on a matter within the scope of
that relationship and while it existed; or […]
(E) was made by the party’s coconspirator during and in furtherance of the
conspiracy.

The statement must be considered but does not by itself establish the declarant’s
authority under (C); the existence or scope of the relationship under (D); or the
existence of the conspiracy or participation in it under (E).
FRE 802
Hearsay is not admissible unless any of the following provides otherwise:
a federal statute;
these rules; or
other rules prescribed by the Supreme Court.
FRE 803
The following are not excluded by the rule against hearsay, regardless of whether the declarant is
available as a witness:
(1) Present Sense Impression. A statement describing or explaining an event or condition, made
while or immediately after the declarant perceived it.
(2) Excited Utterance. A statement relating to a startling event or condition, made while the
declarant was under the stress of excitement that it caused.
(3) Then-Existing Mental, Emotional, or Physical Condition. A statement of the declarant’s then-
existing state of mind (such as motive, intent, or plan) or emotional, sensory, or physical
condition (such as mental feeling, pain, or bodily health), but not including a statement of
memory or belief to prove the fact remembered or believed unless it relates to the validity or
terms of the declarant’s will.
(4) Statement Made for Medical Diagnosis or Treatment. A statement that:
(A) is made for — and is reasonably pertinent to — medical diagnosis or treatment; and
(B) describes medical history; past or present symptoms or sensations; their inception; or
their general cause.
(5) Recorded Recollection. A record that:
(A) is on a matter the witness once knew about but now cannot recall well enough to
testify fully and accurately;
(B) was made or adopted by the witness when the matter was fresh in the witness’s
memory; and
(C) accurately reflects the witness’s knowledge.
If admitted, the record may be read into evidence but may be received as an exhibit only if
offered by an adverse party.
(6) Records of a Regularly Conducted Activity. A record of an act, event, condition, opinion, or
diagnosis if:
(A) the record was made at or near the time by — or from information transmitted by —
someone with knowledge;
(B) the record was kept in the course of a regularly conducted activity of a business,
organization, occupation, or calling, whether or not for profit;
(C) making the record was a regular practice of that activity;
(D) all these conditions are shown by the testimony of the custodian or another qualified
witness, or by a certification that complies with Rule 902(11)  or (12) or with a statute
permitting certification; and
(E)  the opponent does not show that the source of information  or the method or
circumstances of preparation indicate a lack of trustworthiness.
(7) Absence of a Record of a Regularly Conducted Activity. Evidence that a matter is not
included in a record described in paragraph (6) if:
(A) the evidence is admitted to prove that the matter did not occur or exist;
(B) a record was regularly kept for a matter of that kind; and
(C)  the opponent does not show that the possible source of the information  or other
circumstances indicate a lack of trustworthiness.
(8) Public Records. A record or statement of a public office if:
(A) it sets out:
(i) the office’s activities;
(ii) a matter observed while under a legal duty to report, but not including, in a
criminal case, a matter observed by law-enforcement personnel; or
(iii) in a civil case or against the government in a criminal case, factual findings
from a legally authorized investigation; and
(B)  the opponent does not show that the source of information  or other circumstances
indicate a lack of trustworthiness.
(9) Public Records of Vital Statistics. A record of a birth, death, or marriage, if reported to a
public office in accordance with a legal duty.
(10) Absence of a Public Record. Testimony — or a certification under Rule 902 — that a
diligent search failed to disclose a public record or statement if:
(A) the testimony or certification is admitted to prove that
(i) the record or statement does not exist; or
(ii) a matter did not occur or exist, if a public office regularly kept a record or
statement for a matter of that kind; and
(B) in a criminal case, a prosecutor who intends to offer a certification provides written
notice of that intent at least 14 days before trial, and the defendant does not object in
writing within 7 days of receiving the notice — unless the court sets a different time for
the notice or the objection.
(11) Records of Religious Organizations Concerning Personal or Family History. A statement of
birth, legitimacy, ancestry, marriage, divorce, death, relationship by blood or marriage, or similar
facts of personal or family history, contained in a regularly kept record of a religious
organization.
(12) Certificates of Marriage, Baptism, and Similar Ceremonies. A statement of fact contained in
a certificate:
(A) made by a person who is authorized by a religious organization or by law to perform
the act certified;
(B) attesting that the person performed a marriage or similar ceremony or administered a
sacrament; and
(C) purporting to have been issued at the time of the act or within a reasonable time after
it.
(13) Family Records. A statement of fact about personal or family history contained in a family
record, such as a Bible, genealogy, chart, engraving on a ring, inscription on a portrait, or
engraving on an urn or burial marker.
(14) Records of Documents That Affect an Interest in Property. The record of a document that
purports to establish or affect an interest in property if:
(A) the record is admitted to prove the content of the original recorded document, along
with its signing and its delivery by each person who purports to have signed it;
(B) the record is kept in a public office; and
(C) a statute authorizes recording documents of that kind in that office.
(15) Statements in Documents That Affect an Interest in Property. A statement contained in a
document that purports to establish or affect an interest in property if the matter stated was
relevant to the document’s purpose — unless later dealings with the property are inconsistent
with the truth of the statement or the purport of the document.
(16) Statements in Ancient Documents. A statement in a document that was prepared before
January 1, 1998, and whose authenticity is established.
(17) Market Reports and Similar Commercial Publications. Market quotations, lists, directories,
or other compilations that are generally relied on by the public or by persons in particular
occupations.
(18) Statements in Learned Treatises, Periodicals, or Pamphlets. A statement contained in a
treatise, periodical, or pamphlet if:
(A) the statement is called to the attention of an expert witness on cross-examination or
relied on by the expert on direct examination; and
(B) the publication is established as a reliable authority by the expert’s admission or
testimony, by another expert’s testimony, or by judicial notice.
If admitted, the statement may be read into evidence but not received as an exhibit.
(19) Reputation Concerning Personal or Family History. A reputation among a person’s family
by blood, adoption, or marriage — or among a person’s associates or in the community —
concerning the person’s birth, adoption, legitimacy, ancestry, marriage, divorce, death,
relationship by blood, adoption, or marriage, or similar facts of personal or family history.
(20) Reputation Concerning Boundaries or General History. A reputation in a community —
arising before the controversy — concerning boundaries of land in the community or customs
that affect the land, or concerning general historical events important to that community, state, or
nation.
(21) Reputation Concerning Character. A reputation among a person’s associates or in the
community concerning the person’s character.
(22) Judgment of a Previous Conviction. Evidence of a final judgment of conviction if:
(A) the judgment was entered after a trial or guilty plea, but not a nolo contendere plea;
(B) the conviction was for a crime punishable by death or by imprisonment for more than
a year;
(C) the evidence is admitted to prove any fact essential to the judgment; and
(D) when offered by the prosecutor in a criminal case for a purpose other than
impeachment, the judgment was against the defendant.
The pendency of an appeal may be shown but does not affect admissibility.
(23) Judgments Involving Personal, Family, or General History, or a Boundary. A judgment that
is admitted to prove a matter of personal, family, or general history, or boundaries, if the matter:
(A) was essential to the judgment; and
(B) could be proved by evidence of reputation.
(24) [Other Exceptions .] [Transferred to Rule 807 .]
FRE 804
(a) Criteria for Being Unavailable. A declarant is considered to be unavailable as a witness if
the declarant:
(1) is exempted from testifying about the subject matter of the declarant’s statement
because the court rules that a privilege applies;
(2) refuses to testify about the subject matter despite a court order to do so;
(3) testifies to not remembering the subject matter;
(4) cannot be present or testify at the trial or hearing because of death or a then-existing
infirmity, physical illness, or mental illness; or
(5) is absent from the trial or hearing and the statement’s proponent has not been able, by
process or other reasonable means, to procure:
(A) the declarant’s attendance, in the case of a hearsay exception under Rule
804(b)(1) or (6); or
(B) the declarant’s attendance or testimony, in the case of a hearsay exception
under Rule 804(b)(2), (3), or (4).
But this subdivision (a) does not apply if the statement’s proponent procured or
wrongfully caused the declarant’s unavailability as a witness in order to prevent
the declarant from attending or testifying.
(b) The Exceptions. The following are not excluded by the rule against hearsay if the declarant
is unavailable as a witness:
(1) Former Testimony. Testimony that:
(A) was given as a witness at a trial, hearing, or lawful deposition, whether given
during the current proceeding or a different one; and
(B) is now offered against a party who had — or, in a civil case, whose
predecessor in interest had — an opportunity and similar motive to develop it by direct,
cross-, or redirect examination.
(2) Statement Under the Belief of Imminent Death. In a prosecution for homicide or in a
civil case, a statement that the declarant, while believing the declarant’s death to be imminent,
made about its cause or circumstances.
(3) Statement Against Interest. A statement that:
(A) a reasonable person in the declarant’s position would have made only if the
person believed it to be true because, when made, it was so contrary to the
declarant’s proprietary or pecuniary interest or had so great a tendency to
invalidate the declarant’s claim against someone else or to expose the declarant to
civil or criminal liability; and
(B) is supported by corroborating circumstances that clearly indicate its
trustworthiness, if it is offered in a criminal case as one that tends to expose the
declarant to criminal liability.
(4) Statement of Personal or Family History. A statement about:
(A) the declarant’s own birth, adoption, legitimacy, ancestry, marriage, divorce,
relationship by blood, adoption, or marriage, or similar facts of personal or family
history, even though the declarant had no way of acquiring personal knowledge
about that fact; or
(B) another person concerning any of these facts, as well as death, if the declarant
was related to the person by blood, adoption, or marriage or was so intimately
associated with the person’s family that the declarant’s information is likely to be
accurate.
(5) [Other Exceptions .] [Transferred to Rule 807.]
FRE 805
- Hearsay within hearsay is not excluded by the rule against hearsay if each part of he
combined statements conforms with an exception to the rule
FRE 806
- When a hearsay statement — or a statement described in Rule 801(d)(2)(C), (D), or (E)
— has been admitted in evidence, the declarant’s credibility may be attacked, and then
supported, by any evidence that would be admissible for those purposes if the declarant
had testified as a witness. The court may admit evidence of the declarant’s inconsistent
statement or conduct, regardless of when it occurred or whether the declarant had an
opportunity to explain or deny it. If the party against whom the statement was admitted
calls the declarant as a witness, the party may examine the declarant on the statement as
if on cross-examination.
FRE 807
(a) In General. Under the following conditions, a hearsay statement is not excluded by the rule
against hearsay even if the statement is not admissible under a hearsay exception in Rule 803 or
804:
(1) the statement is supported by sufficient guarantees of trustworthiness—after
considering the totality of circumstances under which it was made and evidence, if any,
corroborating the statement; and
(2) it is more probative on the point for which it is offered than any other evidence that
the proponent can obtain through reasonable efforts.
(b) Notice. The statement is admissible only if the proponent gives an adverse party reasonable
notice of the intent to offer the statement—including its substance and the declarant's name—so
that the party has a fair opportunity to meet it. The notice must be provided in writing before the
trial or hearing—or in any form during the trial or hearing of the court, for good cause, excuses a
lack of earlier notice.

Rule 1101. Applicability of the Rules


a) To Courts and Judges. These rules apply to proceedings before:
- United States district courts;
- United States bankruptcy and magistrate judges;
- United States courts of appeals;
- the United States Court of Federal Claims; and
- the district courts of Guam, the Virgin Islands, and the Northern Mariana Islands.
b) To Cases and Proceedings. These rules apply in:
- civil cases and proceedings, including bankruptcy, admiralty, and maritime cases;
- criminal cases and proceedings; and
- contempt proceedings, except those in which the court may act summarily.
c) Rules on Privilege. The rules on privilege apply to all stages of a case or proceeding.
d) Exceptions. These rules — except for those on privilege — do not apply to the
following:
- the court’s determination, under Rule 104(a), on a preliminary question of fact
governing admissibility;
- grand-jury proceedings; and
- miscellaneous proceedings such as:
■ extradition or rendition;
■ issuing an arrest warrant, criminal summons, or search warrant;
■ a preliminary examination in a criminal case;
■ sentencing;
■ granting or revoking probation or supervised release; and
■ considering whether to release on bail or otherwise.
e) Other Statutes and Rules. A federal statute or a rule prescribed by the Supreme Court
may provide for admitting or excluding evidence independently from these rules.

You might also like